Sie sind auf Seite 1von 43

Summer 2008

Prof. Pouncy
Professional Responsibility Outline
Rule 1.1: Competence
A lawyer shall provide competent representation to a client. Competent representation requires the legal knowledge, skill, thoroughness and preparation reasonably
necessary for the representation.

Rule 1.6 Confidentiality of Information


(a)A lawyer shall not reveal information relating to the representation of a client unless the client gives informed consent, the disclosure is impliedly authorized in order
to carry out the representation or the disclosure is permitted by paragraph (b).
(b.)A lawyer may reveal information relating to the representation of a client to the extent the lawyer reasonably believes necessary:
(1.) to prevent reasonably certain death or substantial bodily harm;
(2.) to prevent the client from committing a crime or fraud that is reasonably
certain to result in substantial injury to the financial interest or property of another and in furtherance of which the client has used or is using the lawyer’s
services.
(3.) to prevent, mitigate or rectify substantial injury to the financial interest or property of another that is reasonably certain to result or has resulted from the
client’s commission of a crime or fraud in furtherance of which the client has used the lawyer’s services;
(4.) to secure legal advice about the lawyer’s compliance with these Rules;
(5.) to establish a claim or defense on behalf of the lawyer in a controversy between the lawyer and the client, to establish a defense to a criminal charge or
civil claim against the lawyer based upon conduct in which the client was involved, or to respond to allegations in any proceeding concerning the lawyer’s
representation of the client; or
(6.) to comply with other law or a court order

Rule 4.3: Dealing With Unrepresented Person


In dealing on behalf of a client with a person who is not represented by counsel, a lawyer shall not state or imply that the lawyer is disinterested. When the lawyer
knows or reasonably should know that the unrepresented person misunderstands the lawyer’s role in the matter, the lawyer shall make reasonable efforts to correct the
misunderstanding. The lawyer shall not give legal advice to an unrepresented person, other than the advice to secure counsel, if the lawyer knows or reasonably should
know that the interest of such a person are or have a reasonable possibility of being in conflict with the interest of the client.

Rule 5.2: Responsibilities of a Subordinate Lawyer


(a.) A lawyer is bound by the Rules of Professional Conduct notwithstanding that the lawyer acted at the direction of another person.
(b.) A subordinate lawyer does not violate the Rules of Professional Conduct if that lawyer acts in accordance with a supervisory lawyer’s reasonable resolution of an
arguable question of professional duty.

Rule 8.3: Reporting Professional Misconduct


(a.) A lawyer who knows that another lawyer has committed a violation of the Rules of Professional Conduct that raises a substantial question as to that lawyer’s
honesty, trustworthiness or fitness as a lawyer in other respects, shall inform the appropriate professional authority.
(b.) A lawyer who knows that a judge has committed a violation of applicable rules of judicial conduct that raises a substantial question as to the judge’s fitness for
office shall inform the appropriate authority.
(c.) This Rule does not require disclosure of information otherwise protected by Rule 1.6 or information gained by a lawyer or judge while participating in an approved
lawyer’s assistance program.

Rule 8.4: Misconduct


It is professional misconduct for a lawyer to:
(a.) violate or attempt to violate the Rules of Professional Conduct, knowingly assist or induce another to do so or do so through the acts of another;
(b.)commit a criminal act that reflects adversely on the lawyer’s honesty, trustworthiness or fitness as a lawyer in other respects;
(c.) engage in conduct involving dishonesty, fraud, deceit or misrepresentation;
(d.) engage in conduct that is prejudicial to the administration of justice;
(e.) state or imply an ability to influence improperly a government agency or official or to achieve results by means that violate the Rules of Professional Conduct or
other law; or
(f.) knowingly assist a judge or judicial officer in conduct that is a violation of applicable rules of judicial conduct or to other law.
Chapter 1: Introduction to Professional Responsibility
I. The Foundations of Professional Responsibility
a. The Lawyer as a Fiduciary
i. Preamble to the American Bar Association’s Model Rules of Professional Conduct:
1. Virtually all difficult ethical problems arise from conflict between a lawyer’s responsibilities to
clients, to the legal system and to the lawyer’s own interest in remaining an ethical person while
earning a satisfactory living
ii. Fiduciaries
1. Have a special obligation to care for and to protect the interest of their beneficiaries
iii. Duties owed by attorneys to client, as fiduciaries:
1. Duty of Competence
a. R 1.1
2. Duty of Loyalty
a. R 1.7
b. R 1.9
c. R 1.8(a)
3. Duty of Confidentiality
a. R 1.6
b. The Lawyer as an Officer of the Court Functioning in an Adversarial System of Justice
1
i. A lawyer is an officer of the court functioning in an adversarial system of justice
ii. Components of an Adversarial System of Justice
1. A neutral decisionmaker
2. Competent advocates zealously presenting the positions of each of the interested parties, and
3. Rules of procedure fairly designed to allow the presentation of relevant evidence to the
decisionmaker
iii. Duty to the judicial system
iv. Sworn to uphold the rules of professional conduct
v. Duty to behave as professionals
vi. Other Roles of Lawyers
1. Lawyer as a counselor
a. Lawyer listens and counsels
2. Lawyer as an advocate
a. Not impartial
b. Zealously present the position of client
i. Advocating for the best solution possible under the circumstances
3. Lawyer as Adversary
a. Judicial Proceedings
b. Presents issues of other side
c. Within context of learned profession
c. The Lawyer as a Person with Personal and Financial Interest
i. Lawyer is a person with personal and financial interest
1. Physical, emotional and financial needs
2. Values and beliefs
3. Strengths and weaknesses
ii. Effective lawyer only if mindful of all roles and design a method for balancing or harmonizing the roles
iii. In re Pautler (p. 5)
1. R 4.3
2. R 8.4
3. Criminal defendant negotiating with police, DA steps in to impersonate PD
4. R. 4.3 Dealing with an unrepresented person
a. An attorney must not imply or state that he is disinterested
i. Pautler implied that he was not in an adverse position
b. A lawyer shall make reasonable efforts to correct the misunderstanding
i. Pautler did not take reasonable efforts, or any efforts, to correct the
misunderstanding
c. A lawyer shall not give legal advice to an unrepresented person other then the advice to
secure counsel
5. An attorney must not give legal advice to those we do not represent
6. If you give legal advice, you represent them, you become liable for any bad advice
7. Pautler made gross misrepresentations of:
a. Who he is
b. What he is, and
c. His relationship to the client
8. R 8.4 Misconduct:
a. Misconduct to violate or attempt to violate a rule of professional conduct or held
someone violate or get someone else to do something that would be a violation if you had
done it
9. STANDARD FROM THE CASE: Strict Compliance with the Rules of Professional
Responsibility
10. Pautler argues his motive (to prevent eminent public harm) constitutes justification for violation
of the rules
a. Court rejects the arguments
i. STRUCTURALLY
1. Motive is irrelevant to the determination of whether a rule has been
violated, it may be relevant as a matter of mitigation
2. Individual opinions are irrelevant if rules give a solution to the problem
ii. DOCTRINAL
1. There was no eminent danger to any specified person

2
II. Resolving Tensions in the Lawyer’s Role: Complying with Rules and Standard of Professional Conduct and the Law
Governing Lawyers
a. Professional Rules and Standards: ABA Codes of Ethics, Ethics Advisory Opinions, Specialized Codes, and Practice
Norms
i. Sources of Lawyers Obligations
1. Professional Standards
a. Model Rules of Professional Conduct
i. Tested in this class and MPRE
b. Rules Regulating the Florida Bar
c. Code of Professional Responsibility
i. Fuzzy, outdated
d. Restatement of the Law Governing Lawyers
i. Not the law anywhere
2. General Legal Principles
a. Rules of Practice and Procedure
i. Meritorious Claim in Civil Procedure
ii. Work-Product Doctrine
b. Laws of Evidence
i. Attorney Client Privilege
c. Tort Law
i. Malpractice
3. Special Standards
a. ABA- The Prosecution Function and the Defense Function
b. American Academy of Matrimonial Lawyers
c. Lawyers creed of professionalism
d. American College of Trust and Estate Counsel
4. Morality
ii. The Model Rules are grouped into eight sections
1. Client-Lawyer Relationship
2. Counselor
3. Advocate
4. Transactions with persons other than clients
5. Law firms and associations
6. Public Service
7. Information about legal services
8. Maintaining the integrity of the profession
III. Going Beyond the Rules: Client-Centered Lawyering, Moral Values, Principles of Professionalism, and Other Sources of
Guidance for Lawyers
a. Philosophy of lawyering:
i. A general approach for dealing with difficult questions that lawyers face in the practice of law
1. Develops consistent behavior
b. Client-centered philosophy:
i. Lawyer acts as zealous partisans on behalf of their clients, doing everything possible to enable their client
to prevail in litigation or to obtain their clients’ objectives in nonlitigation matters, except to the extent that
clear rules of professional conduct or legal principles prohibit the lawyer from acting
1. Resolve conflicts in favor of client
ii. Lawyer’s “Principle of Nonaccountability”
1. Lawyers are not legally or morally accountable for their actions
c. Philosophy of Social Values (Officer of the Court)
i. Fiduciary for the system
ii. Resolve doubts in favor of the values associated with the legal profession
d. Personal Morality
i. Does representation conflict with personal morals (can I sleep at night?)
ii. Will take action the lawyer believes to be indicated by the principles of morality even if this action was not
necessarily in the client’s interest
e. Defensive Lawyering
i. Avoiding liability
f. PROBLEM 1.1 (p. 26)
i. R 1.2(b)
ii. R 6.2(c)
iii. Do lawyers have a moral responsibility?
3
1. Yes, but no responsibility for choosing cases or clients and no responsibility for the outcome of
cases
2. Lawyers are responsible for choosing their area of practice and engaging in moral dialogue with
their clients
3. Lawyers should practice moral activism in representation
4. Work to preserve just legal institutions; reforming deficient legal institutions
IV. Issues Facing the Profession: Methods of Regulating Attorney Conduct:
a. Admission to Practice
b. The Disciplinary System
i. PROBLEM 1.2 –
1. Reporting Attorney Misconduct
a. R 8.3 Reporting Misconduct
i. Required to report if we know of a professional responsibility violation that
raises a substantial question as to the lawyer’s fitness to practice
1. Informing a client is not a substitute for the duty to report
2. Not under a duty to report yourself
b. R 1.0(f) Definition of Knowledge:
i. Knowledge is actual knowledge of the facts in question or knowledge inferred
from the circumstances
1. Suspicions do not equal knowledge
c. R 8.4 – Misconduct
i. Violating rules
ii. Criminal acts
iii. Dishonesty
iv. Conduct prejudicial to the administration of justice
1. “Catch-all”
v. Implying the ability to influence an official
vi. Assisting somebody in prohibited conduct
d. R 5.2 – Responsibilities of a Subordinate
i. There must be an arguable question of professional conduct
1. If it is arguable question, is the response by the supervisor reasonable?
ii. Wieder v. Skala (p. 38)
1. Employment relationship between lawyers and a firm is based on the assumption that law will be
practiced based upon ethical standards of the profession; therefore, the employment at-will
doctrine will not insulate firing decision from scrutiny
2. Breach of contract is recognized for attorney fired to whistleblowing
iii. Bohatch v. Butler & Binion (p. 39)
1. Partners owe a fiduciary duty to each other
2. Firm did not breach its fiduciary duty to Bohatch, but is liable for breaching the partnership
agreement
3. Firing for reporting a violation of the rules will be viewed as a wrongful termination
iv. Why do we require lawyers to report misconduct?
1. We are a self-regulated profession
a. All the people participating in the regulation of lawyers are lawyers
2. Lawyers are in the best position to find out (Superior knowledge)
3. Bolsters public trust
v. Exceptions to the Duty to Report R 8.3(c)
1. Does not require disclosure of confidential information without client’s consent
2. Does not require disclosure of information obtained in representation of a lawyer
a. Where a lawyer is retained to represent a lawyer whose professional conduct is in
question
3. Does not require disclosure of information obtained while participating in an approved lawyer
assistance program
4. Lorna Lawyer was a solo practitioner until she hired you as an associate about four months ago.
While filing research memos you came across a closing statement for a personal injury case. The
statement signed by the Client showed a structured settlement in which $100,000 was paid
immediately and $25,000 was payable over each of the next six years. The firm’s fee was paid
fully out of the initial $100,000. You believe the case had been settled for a lump sum and you
have found a letter from the insurance company indicating that it was enclosing its draft for

4
$250,000 as a lump sum settlement of the case. When you ask Lorna about the discrepancy she
fires you.
a. (a) You should find another job and take no further action on the information
discovered.
i. Disregarding duty to report (R 8.3)
b. (b) You should advise the Client to file a malpractice action against Lorna Lawyer.
i. Still have to report for yourself
c. (c) You should tell Lorna Lawyer that you have retained a copy of the client’s file
and suggest she reconsider her decision to fire you.
i. Extortion
d. (d) You should report Lorna Lawyer to the appropriate disciplinary authority.
i. CORRECT R 5.1(c), R 5.2
5. Lorna Lawyer was a solo practitioner until she hired you as an associate about four months ago.
Yesterday you received a call from Cary Client while Lorna was out of town. Cary told you that
she did not understand why her settlement check was so small; although the case settled for
$250,000 Cary received a check for $17,500. You reviewed the file and found the closing
statement for Cary’s personal injury case. The statement, which was signed by Cary, showed a
structured settlement in which $100,000 was paid immediately and $25,000 was payable over each
of the next six years. The firm’s fee was paid fully out of the initial $100,000. You told Cary
about the closing statement and she was adamant that she never signed it and knew nothing about
a structured settlement. You phoned Lorna and she told you that Cary is senile, you should ignore
her, and she [Lorna] would handle the matter when she returned to town.
a. (a) You must report the matter to the appropriate attorney disciplinary authority in
your jurisdiction.
i. CORRECT, supervising attorney said something inconsistent with the duty
under rules (R 1.14)
b. (b) You are not required to report the matter, but you must conduct a discreet
investigation to obtain more information.
i. Are or are not required
c. (c) You may rely on Lorna’s resolution of the matter.
i. If she is senile none of the previous should have happened
d. (d) You may not report the matter because the information is confidential.
i. Exception would say unless you have informed consent
6. Lorna Lawyer was a solo practitioner until she hired you as an associate about four months ago.
Immediately before the trial of a personal injury matter in which you and Lorna represented the
defendant, Lorna told you to have the office’s private investigator drop a flyer in the mail boxes of
prospective jurors. The flyers stated that if the defendant lost this case he would be forced to close
his manufacturing facility, which would result in the loss of 200 jobs. The investigator asked you
if this was legal, and you assured her it was because Lorna asked you to do it. A mistrial was
declared after five jurors contracted viral pneumonia. The next week you viewed a crime show on
TV in which the issue of jurors contact was addressed and you realized that the contact you had
authorized with the jurors was improper. [see R.3.5(c)]
a. a. You should report the matter to the Model State Bar Association.
i. CORRECT
b. b. You should find another job and take no further action on the information
discovered.
i. Not reporting, would violate R 8.3
c. c. You should tell Lorna what you have learned and rely on her determination of
how you should proceed.
i. There is nothing she can say about it, it’s against the law
d. d. You should write a memo to the file documenting that you undertook the action
at Lorna’s direction.
i. Not reporting, would violate R 8.3
7. You are an associate in a law firm representing a Company being sued for polluting the local
water supply. The Plaintiffs have requested the production of any complaints the company
received from residents concerning the water supply. On May 1, 2007 you find a box of
complaints from residents concerning the water but the box was labeled, “DESTROY THE
CONTENTS OF THIS BOX ON JANUARY 1, 2007 PURSUANT TO THE COMPANY’S
DOCUMENT RETENTION POLICY.” You show the box to your supervising partner in the firm
and ask her what to do. She tells you to destroy the contents of the box.

5
a. (a) You may destroy the contents of the box because your supervisor has made a
reasonable resolution of an arguable question of professional responsibility.
i. Cannot destroy
b. (b) If you destroy the contents of the box you have engaged in misconduct.
c. (c) You may destroy the contents of the box because you have been directed to do so
by a supervisor.
i. Cannot destroy
d. (d) If you destroy the contents of the box your supervisor has engaged in misconduct
but you have not.
i. Cannot destroy
Rule 1.0: Terminology
(a.) “Belief” or “believes” denotes that the person involved actually supposed the fact in question to be true. A person’s belief may be inferred from
circumstances.
(b.) “Confirmed in writing,” when used in reference to the informed consent of a person, denotes informed consent that is given in writing by the person or a
writing that a lawyer promptly transmits to the person confirming an oral informed consent. See paragraph (e) for the definition of “informed consent.” If it
is not feasible to obtain or transmit the writing at the time the person gives informed consent, then the lawyer must obtain or transmit it within a reasonable
time thereafter.
(c.) “Firm” or “law firm” denotes a lawyer or lawyers in a law partnership, professional corporation, sole proprietorship or other association authorized to
practice law; or lawyers employed in a legal services organization or the legal department of a corporation.
(d.) “Fraud” or “fraudulent” denotes conduct that is fraudulent under the substantive or procedural law of the applicable jurisdiction and has a purpose to
deceive.
(e.) “Informed consent” denotes the agreement by a person to a proposed course of conduct after the lawyer has communicated adequate information and
explanation about the material risks of and reasonably available alternatives to the proposed course of conduct.
(f.) “Knowingly,” “know”, or “knows” denotes actual knowledge of the fact in question. A person’s knowledge may be inferred from circumstances.
(g.) “Partner” denotes a member of a partnership, a shareholder in a law firm organized as a professional corporation, or a member of an association authorized
to practice law.
(h.) “Reasonable” or “reasonably” when used in relation to conduct by a lawyer denotes the conduct of a reasonably prudent and competent lawyer.
(i.) “Reasonable belief” or “reasonably believes” when used in reference to a lawyer denotes that the lawyer believes the matter in question and that the
circumstances are such that the belief is reasonable.
(j.) “Reasonably should know” when used in reference to a lawyer denotes that a lawyer of reasonable prudence and competence would ascertain the matter in
question.
(k.) “Screened” denotes the isolation of a lawyer from any participation in a matter through the timely imposition of procedures within a firm that are reasonably
adequate under the circumstances to protect information that the isolated lawyer is obligated to protect under these Rules or other laws.
(l.) “Substantial” when used in reference to degree or extent denotes a material matter of clear and weighty importance.
(m.) “Tribunal” denotes a court, an arbitrator in a binding arbitration proceeding or a legislative body, administrative agency or other body acting in an
adjudicative capacity. A legislative body, administrative agency or other body acts in an adjudicative capacity when a neutral official, after the presentation
of evidence or legal argument by a party or parties, will render a binding legal judgment directly affecting a party’s interest in a particular matter.
(n.) “Writing” or “written” denotes a tangible or electronic record of a communication or representation, including handwriting, typewriting, printing,
photostating, photography, audio or video recording and e-mail. A “signed” writing includes an electronic sound, symbol or process attached to or logically
associated with a writing and executed or adopted by a person with the intent to sign the writing.

Rule 1.1: Competence


A lawyer shall provide competent representation to a client. Competent representation requires the legal knowledge, skill, thoroughness and preparation reasonably
necessary for the representation.

Rule 1.2: Scope of Representation and Allocation of Authority Between Client and Lawyer
(a.) Subject to paragraphs (c) and (d), a lawyer shall abide by a client’s decisions concerning the objectives of representation and, as required by Rule 1.4, shall consult
with the client as to the means by which they are to be pursued. A lawyer may take such action on behalf of the client as is impliedly authorized to carry out the
representation. A lawyer shall abide by a client’s decision whether to settle a matter. In a criminal case, the lawyer shall abide by the client’s decision, after consultation
with the lawyer, as to a plea to be entered, whether to waive jury trial and whether the client will testify.
(b.) A lawyer’s representation of a client, including representation by appointment, does not constitute an endorsement of the client’s political, economic, social or
moral views or activities.
(c.) A lawyer may limit the scope of the representation if the limitation is reasonable under the circumstances and the client gives informed consent.
(d.) A lawyer shall not counsel a client to engage, or assist a client, in conduct that the lawyer knows is criminal or fraudulent, but a lawyer may discuss the legal
consequences of any proposed course of conduct with a client and may counsel or assist a client to make a good faith effort to determine the validity, scope, meaning or
application of the law.

Rule 1.6 Confidentiality of Information


(a)A lawyer shall not reveal information relating to the representation of a client unless the client gives informed consent, the disclosure is impliedly authorized in order
to carry out the representation or the disclosure is permitted by paragraph (b).
(b.)A lawyer may reveal information relating to the representation of a client to the extent the lawyer reasonably believes necessary:
(1.) to prevent reasonably certain death or substantial bodily harm;
(2.) to prevent the client from committing a crime or fraud that is reasonably
certain to result in substantial injury to the financial interest or property of another and in furtherance of which the client has used or is using the lawyer’s
services.
(3.) to prevent, mitigate or rectify substantial injury to the financial interest or property of another that is reasonably certain to result or has resulted from the
client’s commission of a crime or fraud in furtherance of which the client has used the lawyer’s services;
(4.) to secure legal advice about the lawyer’s compliance with these Rules;
(5.) to establish a claim or defense on behalf of the lawyer in a controversy between the lawyer and the client, to establish a defense to a criminal charge or
civil claim against the lawyer based upon conduct in which the client was involved, or to respond to allegations in any proceeding concerning the lawyer’s
representation of the client; or
(6.) to comply with other law or a court order

6
Rule 1.7: Conflict of Interest: Current Clients
(a.) Except as provided in paragraph (b), a lawyer shall not represent a client if the representation involves a concurrent conflict of interest. A concurrent conflict of
interest exists if:
(1.) the representation of one client will be directly adverse to another client; or
(2.) there is a significant risk that the representation of one or more clients will be materially limited by the lawyer’s
responsibilities to another client, a former client or a third person or by a personal interest of the lawyer.
(b.) Notwithstanding the existence of a concurrent conflict of interest under paragraph (a), a lawyer may represent a client if:
(1.) the lawyer reasonably believes that the lawyer will be able to provide competent and diligent representation to each
affected client;
(2.) the representation is not prohibited by law;
(3.) the representation does not involve the assertion of a claim by one client against another client represented by the
lawyer in the same litigation or other proceeding before a tribunal; and
(4.) each affected client gives informed consent, confirmed in writing.

Rule 1.14: Client with Diminished Capacity


(a.) When a client’s capacity to make adequately considered decisions in connection with a representation is diminished, whether because of minority, mental
impairments or for some other reason, the lawyer shall, as far as reasonably possible, maintain a normal client-lawyer relationship with the client.
(b.) When the lawyer reasonably believes that the client has diminished capacity, is at risk of substantial physical, financial or other harm unless action is taken and
cannot adequately act in the client’s own interest, the lawyer may take reasonably necessary protective action, including consulting with individuals or entities that have
the ability to take action to protect the client and, in appropriate cases, seeking the appointment of a guardian ad litem, conservator or guardian.
(c.) Information relating to the representation of a client with diminished capacity is protected by Rule 1.6. When taking protective action pursuant to paragraph (b), the
lawyer is impliedly authorized under Rule 1.6(a) to reveal information about the client, but only to the extent reasonably necessary to protect the client’s interest.

Rule 1.16: Declining or Terminating Representation


(a.) Except as stated in paragraph (c), a lawyer shall not represent a client or, where representation has commenced, shall withdraw from the representation of a client if:
(1.) the representation will result in a violation of the rules of professional conduct or other law;
(2.) the lawyer’s physical or mental condition materially impairs the lawyer’s ability to represent the client; or
(3.) the lawyer is discharged
(b.) Except as stated in paragraph (c), a lawyer may withdraw from representing a client if:
(1.) withdrawal can be accomplished without material adverse effect on the interest of the client;
(2.) the client persists in a course of action involving the lawyer’s services that the lawyer reasonably believes is criminal or fraudulent;
(3.) the client has used the lawyer’s services to perpetrate a crime or fraud;
(4.) the client insist upon taking action that the lawyer considers repugnant or with which the lawyer has a fundamental disagreement;
(5.) the client fails substantially to fulfill an obligation to the lawyer regarding the lawyer’s services and has been given reasonable warning that the lawyer
will withdraw unless the obligation is fulfilled;
(6.) the representation will result in an unreasonable financial burden on the lawyer or has been rendered unreasonably difficult by the client; or
(7.) other good cause for withdrawal exists
(c.) A lawyer must comply with applicable law requiring notice to or permission of a tribunal when terminating a representation. When ordered to do so by a tribunal, a
lawyer shall continue representation notwithstanding good cause for terminating the representation.
(d.) Upon termination of representation, a lawyer shall take steps to the extent reasonably practicable to protect a client’s interests, employment of other counsel,
surrendering papers and property to which the client is entitled and refunding any advance payment of fee or expense that has not been earned or incurred. The lawyer
may retain papers relating to the client to the extent permitted by other law.

Rule 3.1: Meritorious Claims and Contentions


A lawyer shall not bring or defend a proceeding, or assert or controvert an issue therein, unless there is a basis in law and fact for doing so that is not frivolous, which
includes a good faith argument for an extension, modification or reversal of existing law. A lawyer for the defendant in a criminal proceeding, or the respondent in a
proceeding that could result in incarceration, may nevertheless so defend the proceeding as to require that every element of the case be established.

Rule 3.3: Candor Toward the Tribunal


(a.) A lawyer shall not knowingly:
(1.) make a false statement of fact or law to a tribunal or fail to correct a false statement of material fact or law previously made to the tribunal by the lawyer.
(2.) fail to disclose to the tribunal legal authority in the controlling jurisdiction known to the lawyer to be directly adverse to the position of the client and not
disclosed by opposing counsel; or
(3.) offer evidence that the lawyer knows to be false. If a lawyer, the lawyer’s client, or a witness called by the lawyer, has offered material evidence and the
lawyer comes to know of its falsity, the lawyer shall take reasonable remedial measures, including, if necessary, disclosure to the tribunal. A lawyer may
refuse to offer evidence, other than the testimony of a defendant in a criminal matter, that the lawyer reasonably believes is false.
(b.) A lawyer who represents a client in an adjudicative proceeding and who knows that a person intends to engage, is engaging or has engaged in criminal or fraudulent
conduct related to the proceeding shall take reasonable remedial measures, including, if necessary, disclosure to the tribunal.
(c.) The duties stated in paragraphs (a) and (b) continue to the conclusion of the proceeding, and apply even if compliance requires disclosure of information otherwise
protected by Rule 1.6.
(d.) In an ex parte proceeding, a lawyer shall inform the tribunal of all material facts known to the lawyer that will enable the tribunal to make an informed decision,
whether or not the facts are adverse.

Rule 3.4: Fairness to Opposing Party and Counsel


A lawyer shall not:
(a.) unlawfully obstruct another party’s access to evidence or unlawfully alter, destroy or conceal a document or other material having potential evidentiary value. A
lawyer shall not counsel or assist another person to do any such act;
(b.) falsify evidence, counsel or assist a witness to testify falsely, or offer an inducement to a witness that is prohibited by law;
(c.) knowingly disobey an obligation under the rules of a tribunal, except for an open refusal based on an assertion that no valid obligation exists;
(d.) in pretrial procedure, make a frivolous discovery request or fail to make reasonably diligent effort to comply with a legally proper discovery request by an opposing
party.
(e.) in trial, allude to any matter that the lawyer does not reasonably believe is relevant or that will not be supported by admissible evidence, assert personal knowledge
of facts in issue except when testifying as a witness, or state a personal opinion as to the justness of a cause, the credibility of a witness, the culpability of a civil litigant
or the guilt or innocence of an accused; or
(f.) request a person other than a client to refrain from voluntarily giving relevant information to another party unless:
(1.) the person is a relative or an employee or other agent of a client; and

7
(2.) the lawyer reasonably believes that the person’s interest will not be adversely affected by refraining from giving such
information.

Rule 3.5: Impartiality and Decorum of the Tribunal


A lawyer shall not:
(a.) seek to influence a judge, juror, prospective juror or other official by means prohibited by law;
(b.) communicate ex parte with such a person during the proceeding unless authorized to do so by law or court order;
(c.) communicate with a juror or prospective juror after discharge of the jury if:
(1.) the communication is prohibited by law or court order;
(2.) the juror has made known to the lawyer a desire not to communicate; or
(3.) the communication involves misrepresentation, coercion, duress or harassment; or
(d.) engage in conduct intended to disrupt a tribunal.

Rule 3.6: Trial Publicity


(a.) A lawyer who is participating or has participated in the investigation or litigation of a matter shall not make an extrajudicial statement that the lawyer knows or
reasonably should know will be disseminated by means of public communication and will have a substantial likelihood of materially prejudicing an adjudicative
proceeding in the matter.
(b.) Notwithstanding paragraph (a), a lawyer may state:
(1.) the claim, offense or defense involved and, except when prohibited by law, the identity of the persons involved;
(2.) information contained in a public record
(3.) than an investigation of a matter is in progress;
(4.) the scheduling or result of any step in litigation
(5.) a request for assistance in obtaining evidence and information necessary thereto;
(6.) a warning of danger concerning the behavior of a person involved, when there is a reason to believe that there exists
the likelihood of substantial harm to an individual or to the public interest, and
(7.) in a criminal case, in addition to subparagraphs (1) through (6):
(i.) the identity, residence, occupation and family status of the accused
(ii.) if the accused has not been apprehended, information necessary to aid in the apprehension of that person;
(iii.) the fact, time and place of arrest; and
(iv.) the identity of investigating and arresting officers or agencies and length of the investigation.
(c.) Notwithstanding paragraph (a), a lawyer may make a statement that a reasonable lawyer would believe is required to protect, a client from the substantial undue
prejudicial effect of recent publicity not initiated by the lawyer or the lawyer’s client. A statement made pursuant to this paragraph shall be limited to such information
as is necessary to mitigate the recent adverse publicity.
(d.) No lawyer associated in a firm or government agency with a lawyer subject to paragraph (a) shall make a statement prohibited by paragraph (a).

Rule 3.8: Special Responsibilities of a Prosecutor


The prosecutor in a criminal case shall:
(a.) refrain from prosecuting a charge the prosecutor knows is not supported by probable cause;
(b.) make reasonable efforts to assure that the accused has been advised of the right to, and the procedure for obtaining, counsel and has been given reasonable
opportunity to obtain counsel.
(c.) not seek to obtain from an unrepresented accused a waiver of important pretrial rights, such as the right to a preliminary hearing;
(d.) make timely disclosures to the defense of all evidence or information known to the prosecutor that tends to negate the guilt of the accused or mitigates the offense,
and, in connection with sentencing, disclose to the defense and to the tribunal all unprivileged mitigating information known to the prosecutor, except when the
prosecutor is relieved of this responsibility by a protective order of the tribunal.
(e.) not subpoena a lawyer in a grand jury or other criminal proceeding to present evidence about a past or present client unless the prosecutor reasonably believes:
(1.) the information sought is not protected from disclosure by any applicable privilege;
(2.) the evidence sought is essential to the successful completion of an ongoing investigation or prosecution; and
(3.) there is no other feasible alternative to obtain the information;
(f.) except for statements that are necessary to inform the public of the nature and extent of the prosecutor’s action and that serve a legitimate law enforcement purpose,
refrain from making extrajudicial comments that have a substantial likelihood of heightening public condemnation of the accused and exercise reasonable care to
prevent investigators, law enforcement personnel, employees or other persons assisting or associated with the prosecutor in a criminal case from making an extrajudicial
statement that the prosecutor would be prohibited from making under Rule 3.6 or this rule.

Rule 4.2: Communication with Person Represented by Counsel


In representing a client, a lawyer shall not communicate about the subject of the representation with a person the lawyer knows to be represented by another lawyer in
the matter, unless the lawyer has the consent of the other lawyer or is authorized to do so by law or a court order.

Rule 4.4: Respect for Rights of Third Persons


(a.) In representing a client, a lawyer shall not use means that have no substantial purpose other than to embarrass, delay, or burden a third person, or use methods of
obtaining evidence that violate the legal rights of such a person.
(b.) A lawyer who receives a document relating to the representation of the lawyer’s client and knows of reasonably should know that the document was inadvertently
sent shall promptly notify the sender.

Rule 8.4: Misconduct


It is professional misconduct for a lawyer to:
(a.) violate or attempt to violate the Rules of Professional Conduct, knowingly assist or induce another to do so or do so through the acts of another;
(b.)commit a criminal act that reflects adversely on the lawyer’s honesty, trustworthiness or fitness as a lawyer in other respects;
(c.) engage in conduct involving dishonesty, fraud, deceit or misrepresentation;
(d.) engage in conduct that is prejudicial to the administration of justice;
(e.) state or imply an ability to influence improperly a government agency or official or to achieve results by means that violate the Rules of Professional Conduct or
other law; or
(f.) knowingly assist a judge or judicial officer in conduct that is a violation of applicable rules of judicial conduct or to other law.

Chapter 2: Defense and Prosecution of Criminal Cases (p. 61)


I. The Client-Lawyer Relationship

8
a. Problem 2-1 (p. 262)
i. Why should we defend the guilty?
1. Presumption of innocence until trial
a. Guilt is not a fact
b. Guilt is a conclusion made by trier of fact
2. Barry Winston, Stranger than True:
a. System is not perfect, the flipped car
3. Innocence Project
a. 217 people exonerated
4. Safeguarding the System
5. Moral teaching to the defendants
6. Horror of prison
7. It’s fun
a. Thrill rush
b. Problem 2-2 (p. 72)
i. Facts: Donald accused of robbing store, convicted, attorney that represented him did a bad job, a case for
post conviction relief?
ii. Ineffective Assistance of Counsel violated Due Process
iii. R 1.1 Competence
1. Competent Representation:
a. Legal knowledge
i. Have it or the ability to obtain it in a reasonable time
b. Skill
c. Thoroughness
d. Preparation
2. In this problem, probably lacking
a. Did not ask question to key witness
b. Told him he couldn’t testify
c. Spent less than an hour with the client, met him one week before trial
3. METHODS OF RECOURSE:
a. Violation of Rule of Professional Responsibility
i. File a grievance
b. If incompetence rises to the level of ineffective assistance of counsel
i. New trial
iv. What is ineffective assistance of counsel
1. Constitutional doctrine protected by the 6th Amendment
2. STRICKLAND TEST: (p.75)
a. Defendant must show that counsel’s performance fell below an objective standard of
“reasonably effective assistance”
i. “Reasonably effective assistance”- What most attorneys would have done in the
situation
b. Counsel’s performance must be prejudicial to the defense in order to constitute
ineffective assistance under the Constitution
i. Reasonable probability that but for the counsel’s errors result would have been
different in a way favorable for the client
ii. WHERE PREJUDICE IS ASSUMED (p. 77-78)
1. Actual Conflict of Interest
a. There is an actual conflict between the lawyer and client when
during the course of the representation the attorney’s and
defendant’s interest diverge with respect to a material fact or
issue or to a course of action.
i. The Conflict yields an actual lapse in representation
when 1.) Plausible defense strategy not undertaken
by counsel, and Strategy not undertaken because of
the conflict.2.) Trial Court knew of the conflict and
did nothing to address it or defense counsel actively
represented conflicting interest
2. Structural Errors that go to the accuracy and reliability of the trial
process
a. Ex.: Denial of Constitutional Rights
9
3. Cronic Exception
a. Counsel entirely fails to subject the prosecutions’ case to
meaningful adversarial testing
b. Ex.: Defense counsel concedes defendant’s guilt without
permission or Defense counsel sleeps during substantial
portions of the trial
3. Should Donald’s conviction be set aside because his counsel did not cross-examine witnesses
a. R 1.2(a) Scope
i. This is a tactical decision, left to attorneys, could be reasonable to not cross-
examine in certain situations
ii. Client sets goals, lawyer decides techniques to achieve goals
iii. Client decisions
1. Whether to accept a settlement offer
2. Whether to waive jury trial
3. Whether the Criminal Defendant Client will testify
4. Williams v. Taylor; Wiggins v. Smith (p. 76)
a. Court gives more consideration to defense counsel actions in capital cases
b. Standard is a little lower in capital cases to reach ineffective assistance of counsel
v. Attorney Malpractice
1. Could Donald bring a claim for malpractice?
a. Defendant has to meet a higher standard, typically defendant must prove actual innocence
by obtaining a reversal of his conviction
b. Must be filed within applicable statute of limitations, even if post conviction relief hasn’t
been obtained
vi. Donald Defendant was charged with armed robbery. Donald wanted to take the stand at trial to testify that
he was with his girlfriend at the time of the robbery. Donald had recently returned to the U.S. after serving
in the Peace Corps and was planning to enroll in Yale’s Divinity School next year. Donald’s girlfriend
corroborated Donald’s story, claiming they rented and watched a videotape that evening. However, the
girlfriend does not remember the video they watched or the video store from which it was rented. Lois
Lawyer told Donald that she would not call him to testify because she believes it is always a bad idea for a
criminal defendant to take the stand. She also refused to call the girlfriend to provide an alibi defense.
Lois Lawyer also failed to cross-examine the sole witness, a 70 year old with poor hearing and who wore
thick glasses. Donald was convicted. Has Lois Lawyer failed to provide effective assistance of counsel?
1. (a) Yes because she refused to call Donald’s girlfriend to testify.
a. Tactical decision, gaps in her memory may speak to credibility (objective/might not be
prejudicial)
2. (b) Yes because she failed to confront the eyewitness.
a. Tactical, like P 2-2
3. (c) Yes because she prevented Donald from exercising his constitutional rights.
a. CORRECT: Constitutional violation  Presumption of prejudice (structural errors)
4. (d) Yes because of the Cronic exception.
a. Cannot say the attorney entirely failed
vii. Donald Defendant was charged with armed robbery. Donald wanted to take the stand at trial to testify that
he was with his girlfriend at the time of the robbery. Donald has never been arrested before. Donald’s
girlfriend is a former nun who works at a hospice providing care to the terminally ill. Donald’s girlfriend
corroborated Donald’s story, claiming they rented and watched videotape that evening. However, the
girlfriend does not remember the video they watched or the video store from which it was rented. Lois
Lawyer told Donald that she would not call the girlfriend to provide an alibi defense because Lois did not
believe her story. Lois Lawyer also failed to cross-examine the sole witness, a 70 year old with poor vision
who claimed to be able to identify Donald based on Donald’s voice alone. Donald was convicted. Has
Lois Lawyer failed to provide effective assistance of counsel?
1. (a) No because the attorney is responsible for determining trial strategy.
a. True; but has nothing to do with effective or ineffective
2. b) Yes because most attorneys would have called Donald’s girlfriend as a witness.
a. CORRECT, no evidence testimony would be unreliable
3. (c) No because she remained awake for the entire trial.
a. No evidence
4. (d) Yes because Donald specifically asked that his girlfriend be called as a witness.
a. Same as A, doesn’t help us with our prongs
c. Problem 2-2(b): Fees in Criminal Context

10
i. Rule 1.5(a) Fees
1. Fee must be reasonable
ii. Types of Fees:
1. General Retainer:
a. A payment to the lawyer for agreeing to take the case or for agreeing to be available to
handle legal matters for the client for a specified period of time
i. Earned when received
ii. Deposited into general revenue
2. Special Retainer:
a. An advanced payment for the client of fees for services to be rendered in the future
i. Earned when services are performed
ii. Initially deposited into a trust account
b. Non-refundable special retainers in criminal cases are unethical and void
3. Flat Fees:
a. Payment of a fixed amount for specific services
4. Contingency Fees:
a. NOT ALLOWED IN CRIMINAL ACTIONS OR DOMESTIC RELATIONS
i. Rule 1.8(d)(2)
5. Expense Deposits:
a. Amount paid to the firm to be applied to future expenses in the client’s case
iii. Fee Duties
1. Reasonable
2. Basis and rate must be communicated to the client
iv. Fee Divisions between lawyers in different firms
1. In proportion to services actually performed
2. Pursuant to a joint representation agreement
a. Referral fees are illegal if not doing anything but referring
3. Client must consent in writing
4. Total fee must be reasonable
d. Client Trust Accounts
i. Duty not to commingle funds
ii. Duty to maintain records
iii. Duty to notify promptly
iv. Duty to deliver promptly
v. Duty to give an accounting
e. Problem 2-2(c) – Conflict of Interest
i. May the lawyers accept a security interest in the client’s media proceeds?
ii. Rule 1.8(d)
1. Media rights as payments vs. security interest to secure payment
2. Media rights as payment are prohibited
a. Prevents lawyers from having a financial interest that may interfere with their
professional judgment
3. Rule is silent as to security interest
a. Would go against the spirit of the rule
i. Bad incentives
b. Client may never get the money
i. Son of Sam Laws
4. Mamie Jones has asked you to represent her in the collection of life insurance proceeds for a
policy on her late husband. Her husband died is his sleep as the result of a massive heart attack.
You have agreed to accept the representation and Mamie Jones has agreed to pay a 33.3%
contingency fee.
a. (a) The fee is permissible if not clearly excessive.
i. Clearly excessive
b. (b) The fee is clearly excessive.
i. CORRECT, what work is he doing? He is not taking any risk, guaranteed
payment
c. (c) The fee is permissible if it is stated in writing and if Mamie Jones was given an
opportunity to consult with other counsel before accepting it.
i. Consulting?
d. (d) Unless the fee is approved by the Court it is clearly excessive.

11
i. Courts don’t approve fees
5. Mamie Jones has asked you to represent her in the collection of life insurance proceeds for a
policy on her late husband who disappeared two years ago in the Bermuda Triangle. You have
agreed to do so and Mamie Jones has agreed to pay to a 33.3% contingency fee.
a. (a) The fee is permissible.
i. CORRECT, Risk may not get paid
b. (b) The fee is clearly excessive.
i. No, lawyer doing substantial work with substantial risk
c. (c) The fee is permissible if the insurance company consents.
i. Not a party to representative
d. (d) Unless the fee is approved by the Court it is clearly excessive.
i. Courts don’t usually get involved
II. Confidentiality (p. 85)
a. Rule 1.6:
i. Three different types
1. Attorney-Client Privilege
a. Law of Evidence
b. Very narrow
2. Client Confidentiality
a. R 1.6
b. Ethical Duty
c. Applies to everything, relatively weak, a lot of exceptions
d. NEVER prevents court from getting evidence
3. Attorney Work Product
a. FRCP 26(b)(3)
b. Primarily applies to documents
b. Problem 2-3
i. RULE 1.2(d): Crime Fraud Exception
1. A lawyer may not assist client in criminal or fraudulent conduct
2. A client’s communications are not privileged when a client is seeking legal advice to enable or
assist the client in committing crime fraud
3. If assisting client in rightful conduct that might have bad consequences it does not meet the crime-
fraud exception
4. If assisting client in wrongful conduct crime-fraud exception is met
a. Rule 1.2(a)
i. Have to abide by the client’s determination of the goals of representation
1. bail = goal
2. cant counsel client to engage in crime
3. cant assist client in committing a crime
ii. Issue: Confidentiality
iii. RULE 1.6
1. When can we reveal?
a. Prevent reasonably certain death or substantial bodily harm
2. Confidentiality is very broad, may not be privileged
a. CAN be forced to reveal confidential information
b. CANNOT be forced to reveal privileged information
iv. When does the attorney client confidentiality attach
1. Client
2. Communicates
3. Confidentially with
4. Counsel (i.e. lawyer or its agents) to obtain
5. Counsel (i.e. legal advice)
v. Assume there is no chance for bail to be granted, do you have a duty to tell the authorities the defendant has
killed the missing woman?
1. NO, Past Crime vs. Future Crime distinction
a. No duty to disclose any past conduct
b. Most instances no duty to disclose future conduct
i. R 1.6
ii. Exceptions – serious crime; to prevent serious bodily injury or death
c. Purcell v. District Attorney for Suffolk District (p. 86):

12
i. Client made threats to burn down apartment, attorney tells police, client arrested, Prosecutor subpoenas
attorney to testify, attorney claims privilege.
ii. I: Whether the defendant’s conversation with counsel is privileged. Can the attorney be made to testify
about that conversation?
iii. Does the crime-fraud exception apply
1. NO
iv. Was the attorney’s advice sought in furtherance of a crime?
1. NO
v. Lawyer CANNOT be compelled to testify because the information is privileged
vi. Why treat the Attorney-Client Privilege and R 1.6 Different?
1. Ability to prevent harm, while preserving the attorney client relationship
vii. PURCELL HYPO:
1. During the meeting with the client the lawyer notices that the client is carrying a book entitled
“Arson for Fun and Profit” and the lawyer can see that the client’s shopping bag contains several
cans of lighter fluid, a number of lamp wicks and a number of boxes of matches.
2. The client tells the lawyer “they will get theirs.”
a. May the lawyer reveal the information under R 1.6?
i. YES, there is a basis to reasonably believe, certainty is not required
viii. Must analyze under Rule 1.6(b)(6)
1. Must you disclose under other law?
d. PROBLEM:
i. Defendant tells you that he plans to escape from jail. What are your ethical responsibilities?
1. Ask questions to form a reasonable belief
a. Is he serious?
b. Does he have the capacity to do this?
ii. Lawyer does nothing, defendant tries to escape, a guard is killed. Is the lawyer responsible, liable?
1. NO, not responsible
2. Is he liable?
a. If the victim is identifiable he could be liable under tort law (Tarasoff v. Regents of the
Universtiy of California p. 93)
e. Debbie Defendant is in custody after being charged with aggravated animal cruelty in the death of three dogs, a
felony. During your interview with Debbie you learn that she has an additional six dogs locked in a barn on a
secluded farm outside of town and that the dogs are either dead or starving to death.
i. (a) You must inform the animal protection authorities about the dogs that have been locked in the
barn.
1. Never mandatory
ii. (b) You may inform the animal protection authorities about the dogs that have been locked in the
barn
1. Rule did not contemplate anything other than death or severe bodily harm
iii. (c) You may not inform the animal protection authorities about the dogs that have been locked in the
barn.
1. CORRECT, Dogs are not humans, confidential information
iv. (d) You should have a veterinarian visit the farm and euthanize the dogs.
v. DISTINGUISH THIS PROBLEM FROM McClure v. Thompson
1. Attorney had reasonable belief children were alive, could reveal if they were children.
f. Debbie Defendant is in custody after being charged with aggravated animal cruelty in the death of three dogs, a
felony. Jerry Jones was the eyewitness who reported Debbie to the authorities. Debbie has asked you to apply for
bail so she can “make sure that Jerry loses his memory.” Assuming you have no obligations under the Tarasoff
doctrine:
i. a. You must warn Jerry or withdraw.
1. Not mandatory R 1.6
ii. b. You may not warn Jerry.
1. Question about bail
iii. c. You must apply for bail or withdraw.
1. CORRECT, Client decides course of action (R 1.2), right act with a wrongful consequence
iv. d. You may not apply for bail.
1. Why? Client determines objectives
g. Lawyer represents client in a harassment action involving Client’s landlord. Client calls Lawyer at home in the
middle of the night and tells Lawyer that she has just shot her landlord in the foot. Lawyer tells Client to call for an
ambulance but Client refuses.

13
i. (a) Lawyer must call for an ambulance for Client’s landlord.
1. Never mandatory
ii. (b) Lawyer may call for an ambulance for Client’s landlord.
1. CORRECT, May
iii. (c) If Lawyer call for an ambulance for Client's landlord, Lawyer has violated Client’s
confidentiality and will be subject to discipline.
1. No, while atty has violated confidentiality, there is exception to prevent [further] serious bodily
injury
iv. (d) If Lawyer does not call for an ambulance for Client's landlord and the landlord dies as result of
the gunshot, Lawyer has violated R.1.2(d).
1. No conspiracy under R 1.2(d)
h. Problem 2-4 (p. 99): Dealing with Physical Evidence, Fruit, and Instrumentalities of Crimes
i. I: Do you have to reveal that you know where the money is?
ii. Rule 3.4
1. As officers of the court attorneys must preserve and give evidence to the appropriate authorities
2. Do we have a duty to tell the authorities where the money is?
a. NO, not duty disclose under R 1.6, may be privileged
3. Do you have an obligation to tell the client to turn the money over to the authorities?
a. Advise client to comply with the law
i. R 1.2(d) and R 2.1 state we must advise the client to comply with the law
1. Client refuses then what?
a. Depends on your philosophy of lawyering
4. Cannot advise client to move the money
a. Would violate R 3.4
5. Can you take possession of money?
a. Yes, but then required to turn it in after a reasonable time for inspection
6. HYPO:
a. Defendant’s girlfriend shows up at your office with the money, what will you do?
i. R 4.3 Dealing with an Unrepresented Person
1. Tell her to get an attorney
i. Physical Evidence, Fruits, and Instrumentalities of the crime
i. In re Rider (p. 100)
1. It is an abuse of a lawyer’s professional responsibility knowingly to take possession of and secrete
the fruits and instrumentalities of a crime.
ii. If the lawyer takes possession of evidence in a criminal, lawyer has an obligation to provide the evidence to
the authorities without changing its character, after he has had a reasonable opportunity to inspect it
iii. Doris Defendant is in custody having been charged with the robbery of a jewelry store. Doris told you that
she has hidden the jewelry under the floorboards in her mother’s attic.
1. (1) You should do nothing with regard to this information.
2. (2) You should advise Doris to have the jewelry moved to a more secure location.
3. (3) You should go to the mother’s home and take possession of the jewelry until the
proceedings against Doris have concluded.
4. (4) You should advise Doris to have her mother carpet the attic.
iv. Before you have an opportunity to make a decision with respect to the information in the previous question
Doris’ mother visits your office with a shopping bag filled with the stolen jewelry. Doris’ mother asks you
what she should do with the Jewelry.
1. (1) You should say, “I can’t answer that question, I’m sorry.”
2. (2) You should take possession of the jewelry and tell Doris’ mother not to tell anyone about
it.
3. (3) You should tell Doris’ mother that she needs an attorney.
4. (4) You should tell Doris’ mother to take the jewelry to the police.
v. HYPO:
1. Your client is an engineer. A bridge she designed and constructed collapses. The next day she
brings all of her records concerning the construction of the bridge to your office for “safekeeping”.
What will you do?
a. Ask why the client has brought them to you
b. Tell her they may not be protected
c. If a criminal proceedings brought  Obligation to turn over
d. If tort charges brought  Wait until requested
14
e. What arguments would allow you to retain and prevent investigators from obtaining?
i. Attorney-Client:
1. NO, not communication
ii. Are these documents self-incriminating?
1. Are they testimonial?
a. Is the information such that it would make statements about
issues being determined by the proceeding?
b.
j. Self Incrimination:
i. Protects persons from being compelled to give incriminating testimony
ii. When can the attorney assert attorney-client privilege as to the documents provided to the attorney by the
client?
1. Attorney can assert the privilege if client would be able to assert the privilege against self-
incrimination had the client retained possession of the document
2. Are docs incriminating?
3. Are docs testimonial? If doc makes statement that would be admissible in proceeding
iii. Act of Production Doctrine (United States v. Doe (p. 106))
1. The court found that the act of production would compel respondent to "admit that the records
exist, that they are in his possession, and that they are authentic." Ibid. While not ruling out the
possibility that the Government could devise a way to ensure that the act of turning over the
documents would not incriminate respondent, the court held that the Government had not made
such a showing.
iv. Delroy visits you at your offices and tells you that he expects to be arrested for stealing an ATM from a
strip mall. Delroy also tells you that he left the ATM in a garbage dump outside of town. You accepted
Delroy as a client and you send your investigator to the garbage dump to see if the ATM is there. The
investigator sees the ATM and takes photographs of it. While doing so she slips and falls and as a result of
the fall the ATM is covered with additional garbage and is concealed. As your investigator is leaving the
garbage dump a dump truck arrives and deposits approximately two tons on garbage over the area in which
the ATM is located.
1. (a) If Delroy is arrested you should inform the police of the location of the ATM.
a. No basis for disclosure
2. (b) If you inform the police of the location of the ATM you have violated your duty of
confidentiality to Delroy.
a. True but not best response
3. (c) If Delroy is arrested you should withdraw and advise him to seek other counsel.
a. CORRECT, Conflict of Interest  Concealing evidence = witness
4. (d) If Delroy is arrested you must provide the photographs of the ATM to the police.
a. Photos are not the evidence
v. Debbie Defendant is in custody after being charged with assaulting her landlord. During your interview
with Debbie you learn that she has hidden a video camera in her landlord’s bedroom. The video camera is
connected to a computer which has been programmed to broadcast images from the Landlord’s bedroom
every night between the hours of 8:00 pm and 8:00 am. Debbie tells you “just wait until the world sees
what he and the preacher’s wife are up to almost every night.”
1. (a) You must inform the landlord of the existence of the video camera.
a. Not mandatory
2. (b) You may inform the landlord of the existence of the video camera.
a. No exception R 1.6(b) would permit disclosure
3. (c) You must discretely arrange to have the video camera removed.
a. Not mandatory, would break law
4. (d) You must treat the information as confidential even if it means you’re your client will be
charged with an additional crime.
a. CORRECT, Confidential information, no exception met; shall not disclose
k. PROBLEM 2-5 False Testimony in Criminal Cases (p. 109)
i. I: Perjury
ii. R 3.3 Candor towards the tribunal
1. Candor v. Defendant’s right to testify (R 1.2(a))
a. RULE 1.2(a)
i. Defendant has absolute right to testify
b. R 3.3 Comment 9
i. Don’t let him testify, but there are 5th Amendment issues
15
iii. What will you do?
1. Counsel client against committing perjury
2. You must testify truthfully
3. Your testimony may present inconsistencies with other testimony that might lead the jury to
believe that you are lying
a. Don’t ask client Qs that will cause him to commit perjury
4. State’s case is weak you might get off without testifying
5. If convicted the judge can take perjury into account when setting your sentence
iv. Nix v. Whiteside (p. 120)
1. Lawyer convinces client not to perjure themselves through a threat to tell the court and/or
withdraw
2. The Court said this is ok, because there is no right to commit perjury
a. Therefore no right impeded – therefore no violation of 5th Amendment or ineffective
assistance of counsel
v. Approaches when the defendant still wants to testify
1. Full Representation, i.e. do nothing
2. Disclose to the Court
3. Withdrawal without disclosure
4. Elicit Narrative Testimony
5. Avoidance of knowledge
6. Remonstrate w/ client
vi. When do we know that our client intends to commit perjury?
1. Clearly stated intentions
2. Firm factual basis
3. Independent verification
i. Forming knowledge beyond a reasonable doubt
vii. Do we have a duty to investigate?
1. NO, it is discretionary depending on whether it is helpful to the client
viii. Client demands you call his girlfriend as an alibi witness, what do you do?
1. Don’t call them if you reasonably believe false, she is not a client
a. Rule 3.3(a)(3)
2. If the client insists, still don’t call
a. Rule 1.2 Tactical Decision
ix. What can the client do if the attorney refuses to call his g/f?
1. Fire the attorney
x. If we decide to disclose:
1. Judges options:
a. Declare a mistrial
b. Warn defendant that perjury is a crime
c. Give a jury instruction (R 3.3 Comment 10)
d. Direct defense counsel to use the narrative
e. Do nothing
2. If bench trial:
a. Judge would have to withdraw from the case
xi. Defendant insist on testifying, lawyer discloses, what does the defendant do afterwards?
1. Defendant argues lawyer has violated his privilege against self incrimination
a. Fisher Standard:
i. Information subject to Attorney-Client Privilege
1. NO, 1.2(d) exception
ii. Would the client be able to assert the privilege against self incrimination?
1. If yes to both, then there is ineffective assistance of counsel
l. Withdrawal
i. R 1.16
1. Mandatory Withdrawal
a. When the representation will result in the violation of the Rules of Professional Conduct
or other law
b. Lawyer incompetence
i. Lawyers physical or mental condition materially impairs his ability to represent
c. Termination
16
2.
Permissive Withdrawal
a. Good cause (R 1.16(b)(1) – (b)(7)
m. Anatomy of a Murder
i. “The Lecture”
1. Encouraging the client to remember the facts in a way that supports a particular legal argument
instead of remembering the facts in the way they actually occurred
2. Is the lecture permissible?
a. Creating false testimony v. educating the client as to the legal significance of facts
i. If creating false testimony
1. NO, under R 3.3 and R 3.4(b)
ii. Educating a client
1. Is allowed

ii. Larry Lawyer has been assigned to represent Dora Defendant who has been accused a bludgeoning her
husband and his girlfriend to death in the girlfriend’s bedroom. Dora tells Larry that she had been feeling
ill in the days immediately before the incident. Larry told Dora that she wasn’t feeling ill, instead, she was
feeling as if she were possessed, she was hearing voices that told her to hurt herself and other people, and
all of the television shows she watched were relating the details of her life. He concluded with, “that is the
way you were feeling, isn’t it?
1. (a) Dora has diminished capacity; Larry should seek the appointment of a guardian before
he provides Dora with any advice.
a. She hasn’t said anything to conclude that
2. (b) If Dora wants to testify that she heard voices, felt possessed and heard the details of her
life broadcast on the television Larry must allow Dora to testify even if he reasonably
believes her testimony to be false.
a. CORRECT, Larry has to let her, doesn’t “know” that didn’t happen
3. (c) Defense counsel are provided with wide latitude in representing the criminal defendant;
Larry’s advice is proper.
4. (d) Larry is developing an alternative theory of the case and his advice is proper.
a. Not developing an alternative theory
III. Conflict of Interest
a. Rule 1.14
i. What is diminished capacity?
1. When a client is unable to make decisions that serve their interest
2. Attorney is not looking at goals of client, but the decision process used to reach that goal
b. Problem 2-6 (p. 121)
i. Determining capacity
1. Defendant is incompetent if he lacks capacity to understand the proceedings against him or to
assist in his own defense
a. If competent then need to follow client’s goals (Rule 1.2)
ii. Insanity:
1. Lacks a substantial capacity to appreciate the criminality of her conduct or to conform her conduct
to the requirements of the law
2. Not criminally responsible
iii. Rule 1.14
1. Is client impaired or incompetent?
2. Can client adequately act in client’s own interest?
3. Can the client make legally binding decisions?
4. Are the able to communicate, understand and deliberate?
iv. If diminished capacity still maintain a normal relationship as long as they’re not incompetent
v. If incompetent it is an unavoidable difficult position
1. ABA Mental Health Standards – Counsel must seek competency hearing (it is not a model rule)
vi. Assessing Client Capacity:
1. Client’s ability to articulate reasons for their decision
2. Ability to appreciate consequences of a decision
3. Variability of state of mind
4. Fairness of the decision
5. Consistency of decision with clients known commitments and values
vii. If Impaired-
1. Is the client’s decision “knowing and intelligent?”

17
viii. If Incompetent-
1. Is client’s assistance necessary at trial?
ix. Options for the attorney
1. Standard Representation
2. Seek a Guardian
3. Next of Kin as Proxy Decision maker
4. Act as De Facto Guardian
a. Not permitted by the rules (R 1.2)
5. Persuade Client to Lawyer’s View of Client’s Best Interests
6. Withdraw
x. Client found competent, she wants to enter a guilty plea, what do you do?
1. Move to withdraw under R 1.16(b)(4)
2. Make a record that the plea is contrary to your advice
xi. Court accepts plea, client instructs attorney not to offer evidence in mitigation, what do you do?
1. R 1.2 goals vs. strategies
a. Arguments at sentencing is strategy
xii. Client instructs you not to file an appeal, what do you do?
1. Client decision
xiii. Rule 1.14
1. Attempt to maintain normal attorney client relationship
2. Lawyer may take reasonable protective action to protect the client’s personal financial or other
interest
3. Comment 3
a. Having family members or others participate in the lawyer’s decisions with the client will
not destroy attorney client privilege
xiv. Gary Gilmore Case (p. 138)
1. Fired lawyer cannot file an appeal
xv. After Stanley is convicted of murder, he instructs his attorney not to file any appeals on his behalf even
though the attorney believes that several instances of reversible error took place during trial. Stanley also
instructs his attorney to make no arguments at the sentencing hearing even though the attorney believes that
she can construct a good argument based on Stanley’s recent return from combat. The attorney tells
Stanley that if she makes the argument she can almost guarantee that Stanley will not receive the death
penalty. However, if she is not permitted to make the argument, she can almost guarantee that Stanley will
receive the death penalty. Stanley instructs the attorney to make no arguments at the sentencing hearing.
1. (a) If Stanley is competent, the lawyer must follow Stanley's instructions.
a. Only with respect to goals
2. (b) If Stanley is not criminally responsible, the lawyer may follow Stanley’s instructions.
a. If he’s insane, you may follow? Makes no sense
3. (c) If Stanley is competent, the lawyer may disregard Stanley’s instructions and make the
mitigating argument.
a. CORRECT, Arguments are within the scope of the lawyer’s conduct
4. (d) If the lawyer fails to follow Stanley’s instructions and makes the argument she will be
committing misconduct.
a. Argument is up to the lawyer
c. Problem 2-7: Multiple Representation of Codefendants (p. 139)
i. I: Conflict of interest
1. Because both are charged with the same crime and may not share equal criminality
a. This limits the attorneys availability of arguments to raise
b. Also could raise ineffective assistance of counsel, post conviction
2. Rule 1.7
a. Shall not represent if:
i. Direct adversity
1. Representation of one client is directly adverse to the other client
ii. Material limitation
1. Material ability to represent one limited by representation of the other
b. Types of Conflicts:
i. Concurrent Conflict
1. Direct adversity, or
2. Material limitation
ii. Consentable Conflicts

18
1. Lawyer’s Reasonable Belief
2. All clients give informed consent
iii. Nonconsentable Conflicts
1. Prohibited by law
2. Adverse Claims
ii. Types of Conflicts:
1. Adverse Representation against a Current Clients
2. Multiple Representation of clients in a single matter
3. Representation against a Former Clients
4. Advocate - Witness Conflict
5. Client vs. Lawyer’s personal interests
iii. Why do defendants want multiple representation?
1. Know what other defendant is doing
2. Cheaper
3. Check and control on each other
4. United front = harder job for the prosecutor
iv. What risks are presented
1. Defendants lose representation if conflict arises
2. No attorney client privilege between the clients themselves
v. May a federal judge appoint one counsel for multiple defendants?
1. Judge must first inquire as to whether a conflict exist
2. If there is a conflict, the second prong of Strckland is presumed to have been met
vi. What is defendants hire an attorney together?
1. Judges can reject defendants’ waivers of conflict
2. Defendants must prove an actual conflict existed to later make a claim of ineffective assistance of
counsel
vii. Multiple Representation:
1. Methodology
a. 1. Is there an actual conflict?
i. 2. If yes, then multiple representation is probably inappropriate. The standard is
the lawyer’s reasonable belief that she will be able to provide competent and
diligent representation to each client.
ii. 3. If not, if there are only potential conflicts, and lawyer reasonably believes that
she will be able to provide competent and diligent representation to each client;
and the clients provide informed consent after full disclosure, the representation
may be proper.
2. Prerequisites to Multiple Representation
a. Full Disclosure
i. Advantages
1. Fee reduction
2. Expense sharing
3. Efficiency
4. United front
ii. Disadvantages
1. Actual conflicts may arise
2. Attorney must withdraw
3. No Attorney-Client privilege if disputes develop between jointly
represented parties
iii. Alternatives
1. Ex.: Joint Defense Agreement
a. In writing
b. Court approval
c. State no attorney-client relationship with codefendants
d. Conditional waiver regarding cross-examination
e. Permits withdrawal
i. Once this occurs cross-examination can occur
3. What happens upon conviction in multiple representation cases?
a. Defendant will argue ineffective assistance of counsel
i. Multiple Representation was inappropriate because:
1. He didn’t understand risks
19
2. There were conflicts
ii. That’s why informed consent is important:
1. Rule 1.0(e) Informed Consent:
a. Informed consent denotes the agreement by a person to a
proposed course of conduct after the lawyer has
communicated adequate information and explanation about
the material risks of and reasonably available alternatives to
the proposed course of conduct
4. Jack and John are identical twin brothers who have been charged with credit card fraud. Jack and
John each have one prior conviction for credit card fraud. Jack and John have asked you to
represent both of them on current credit card fraud charge.
a. (a) The representation is proper because there is no serious conflict of interest.
i. B is more complete
b. (b) The representation is proper if counsel believes she can provide competent and
diligent representation to each client and each client gives his informed consent in
writing.
i. CORRECT
c. (c) The representation is not proper because an actual conflict of interest may
develop.
i. No indication of potential conflict
d. (d) The representation is not proper because Jack and John’s interests are adverse.
i. Interest are not adverse
5. Jack and John are identical twin brothers who were injured in an automobile accident. Jack was
driving and John was the passenger. Hospital records indicate that John had been using cocaine
immediately before the accident. Department of Motor Vehicle records indicate that Jack’s
driver’s license was suspended at the time of the accident. Jack and John have asked you to
represent both of them and file a negligence action against the SUV driver who rear-ended them
while they were stopped at a stop light waiting for the light to change.
a. (a) The representation is proper because there is no serious conflict of interest.
b. (b) The representation is proper if counsel believes she can provide competent and
diligent representation to each client and each client gives his informed consent in
writing.
i. CORRECT, Cocaine doesn’t contribute; full standard of R 1.7(b)
c. (c) The representation is not proper because there is an actual conflict of interest.
i. Sitting there does not equal contributory negligence
d. (d) The representation is not proper because Jack and John are identical twins.
6. Jack and John are identical twin brothers who are accused of robbing a fast food store. During the
robbery they are alleged to have worn identical clothing and one is alleged to have pistol whipped
the store’s manager. Jack and John have asked you to defend both of them of the charge of
aggravated robbery.
a. (a) The representation is proper because there is no serious conflict of interest.
b. (b) The representation is proper if counsel believes she can provide competent and
diligent representation to each client and each client gives his informed consent in
writing.
c. (c) The representation is not proper because this is a criminal prosecution.
i. Vague, situations where it could be proper
d. (d) The representation is not proper because Jack and John are identical twins.
i. CORRECT, two identical people; witness can’t decide; lawyer has to argue one
or the other did it
1. Not in the same position, one did something the other didn’t
d. Mickens v. Taylor (p. 147)
i. Attorney was appointed to represent a murder who murdered one of the attorneys assigned clients
ii. I: Did the appeals court err in holding that a defendant must show an actual conflict of interest and an
adverse effect in order to establish a 6th Amendment violation where a trial court fails to inquire into a
potential conflict of interest about which it reasonably should have known?
Is prejudice assumed, yielding an automatic reversal?
iii. Strickland Test:
1. Reasonably effective assistance
2. Second Prong can be established by:
a. Extreme denial of counsel (Cronic)

20
i. Prejudice is presumed and reversal is automatic
b. Reversal if the conflict actually effected the adequacy of the representation
c. Actual prejudice
iv. SCOTUS holds automatic reversal only becomes available when defense counsel objects to the
representation and the judge fails to inquire into the objection
v. Because defense counsel did not object, defendant must show the conflict actually effected the
representation
IV. Limitations on Litigation Tactics by the Prosecution and by the Defense
a. Problem 2-8: Trial Publicity
i. What factors should be taken into consideration?
1. Ethical Considerations – R 3.6
a. Prohibits statements that have a substantial likelihood of materially prejudicing and
adjudicative proceeding in the matter
b. R 3.6(c) – Right to reply
i. An attorney can respond if the other attorney’s statement has had a prejudicial
statement
ii. Comment 5 is the standard for material prejudice
1. Character, credibility, reputation, criminal record, plea negotiations,
confessions, refusals to make statements, test results, physical
evidence, opinion as to guilt or innocence, inadmissible information,
the fact that the defendant has been charged without a statement of the
presumption of innocence.
2. Tactical Considerations
a. What is my client’s interest? Need to respond? How should we respond? When?
ii. Should the defense counsel consult with the client before making a response?
1. R 1.2
a. Don’t have to, a tactical decision
2. But probably should talk to client because it could damage the attorney-client relationship
iii. Gentile v. Nevada (p. 156)
1. Overly detailed response
iv. What should we do about the prosecutor’s ethical violations (none of these very helpful)
1. Contempt (would req ct order, e.g., gag order, first & THEN prosecutor’s violation thereof)
2. Move to dismiss
3. File disciplinary complaint (but, not going to help client)
4. Instruct the jury to ignore the pre-trial publicity
v. Duke Lacrosse Case:
1. Rule 3.8(f)
a. Applies 3.6 to the Prosecutors agents (which includes the police)
vi. After Martha’s arrest for the drowning death of her husband, the prosecutor gives a press conference in
which she reports that Martha’s 8-year-old niece saw Martha hit her husband with an oar before he sunk
under the water. You represent Martha and believe that Martha’s niece actually saw Martha extending the
oar to her husband in an attempt to rescue him.
1. (a) You must immediately call a new conference to rebut the prosecutor’s story.
a. Never must
2. (b) You may only rebut the prosecutor’s statements if they were made in bad faith.
a. Nothing about bad faith
3. (c) The prosecutor’s actions are improper as they are likely to prejudice Martha’s ability to
get a fair trial.
a. CORRECT, R 3.6
4. (d) The prosecutor’s actions are improper because they are vindictive.
a. N/a – vindictive goes to why bringing action
vii. Problem 2-8 (b)
1. Is the Gag order valid?
a. Standard (US v. Salameh):
i. No broader than necessary to protect the integrity of the judicial system and the
defendant's right to a fair trial
ii. Explore other available remedies (least restrictive means)
2. R 8.2: Judicial and Legal Officials
a. Criticizing judges

21
i. Were the statements false or made with a reckless disregard to their
truthfulness?
1. If so, they violated the rule
b. Problem 2-9: Limitation on Trial Tactics
i. Whether testimony of the b/f and frat brothers about credibility is inadmissible because of rape shield law
1. R 3.4(e)
a. May not allude to any matter that the lawyer does not reasonably believe is relevant or
that will not be supported by admissible evidence
ii. If the strategy is permissible must the defense counsel use it?
1. If not, did the attorney deny effective assistance of counsel
a. Strickland
i. Would attorneys in jurisdiction engage in this activity
ii. Would the outcome likely be different
iii. Is it ok to place b/f and frat bros on the witness list?
1. Rule 4.4(a) Respect for Third Parties
a. Is the purpose of calling the witnesses just to embarrass or burden
b. In criminal cases a VICTIM is a THIRD PARTY
iv. “Set-up” theory
1. Can the defense pursue a defense based on a manufactured theory?
a. Case law seems to support in criminal cases to test whether there is a reasonable doubt
2. Is lawyer generated theory proper under R 3.4(e)
a. In criminal cases yes, civil cases probably not (FRCP 11)
v. HYPO:
1. After defense files its motion under the rape shield law 7th Circuit holds that the rape shield law
does not violate the Confrontation Clause.
a. R 3.3(a)(2)
i. Case must be:
1. Controlling, and
2. Directly adverse
ii. Must defense reveal?
1. No, federal courts not controlling we are in State Court (unless it’s a
SCOTUS case)
2. Prosecution finds a case Olden, where defendant's 6th amendment rights were found to be violated
when the victim could not be questioned about her cohabitation.
a. Does prosecution have to reveal?
i. No, not directly adverse
3. No duty to reveal adverse fact, but must reveal authority
vi. Duty to Disclose:
1. R 3.4(d) – discovery rules
2. R 8.4(b), (c) – disclosure to prevent fraud
3. R 3.3(a)(2) – adverse law
4. R 3.3(a)(3) – disclosure of false statement of material fact to tribunal to prevent perjury
5. R 3.3(b) – duty to disclose to remedy perjury
6. R 3.3(d) – disclosure in ex parte proceedings
c. Problem 2.9: Is the investigation of potential jurors proper?
i. R 3.5
1. Prohibits vexatious or harassing conduct
2. Improper to influence a juror or prospective juror
3. Improper to communicate ex parte with a juror or potential juror
4. Can engage in proceedings unlikely to come to a jurors attention
ii. Credit reports and drive by not improper
iii. Interviews may be improper, because it is likely someone would convey that information to the juror
iv. Discriminatory Preemptive Challenges
1. Batson Test
2. strike someone based on race, gender or ethnicity
v. Does the defense have a duty to disclose that the juror is employed by a subsidiary of McSwain
Enterprises?
1. R 3.3(b) Jury Misconduct
a. Does the defense counsel have discretion to disclose?
22
i. Yes, under R 1.2(a) it’s a strategic action
vi. Trial results in a hung jury can defense interview former jurors?
1. R 3.5-
a. Yes, as long as juror consents and not prohibited by law
vii. You are defending Robert Ruiz against an embezzlement charge. Lisa Lawyer is assisting you in defending
Ruiz. Yesterday, Lisa was on her way to the courthouse when her car broke down. Judy, a passerby
stopped and asked if she could help. Lisa said she was in hurry to get to the courthouse and Judy said she
was going that way and agreed to drive Lisa. It wasn’t until Lisa was in the courtroom that she realized
that Judy was one of the jurors hearing the case against Ruiz.
1. (a) Lisa and Judy did not discuss the case so Lisa’s conduct was not improper.
a. Doesn’t matter
2. (b) Lisa was not aware that Judy was a juror in the case so Lisa’s conduct was not improper.
a. No knowledge requirement
3. (c) If Lisa had been the lead attorney instead of an assisting attorney her conduct would
have been improper.
a. Doesn’t matter; lawyers represent client, doesn’t matter if they divide
4. (d) Even though Lisa and Judy did not discuss the case Lisa’s conduct was improper.
a. CORRECT

d. Document Retention
i. If there are proceedings or future proceedings where the documents would be relevant you cannot destroy
them, they have substantial evidentiary value
e. Witness Examination
i. Should you cross examine a truthful witness?
1. R 4.4(a)
a. Permits, because a lawyer engaging has a substantial purpose other than harming the
interest, making the prosecutor prove his case beyond a reasonable doubt
b. Should because of proof beyond a reasonable doubt, questioning could raise questions or
raise question of credibility
c. Your determination of whether witness in truthful doesn’t matter
i. Should think about how you do it
ii. Different if you are prosecutor
f. Problem 2-10: Special Duties of Prosecutors
i. To do justice!!!!
1. Rule 3.8 (Ethical Duty)
2. Brady Material (Constitutional Duty)
3. Prosecutorial Discretion
4. Ex parte contacts with defendants
5. Trial Conduct
ii. Is there a duty for the prosecutor to turn over information about defendant misidentification to the grand
jury
1. No, it’s a matter of discretion, U.S. v. Williams (p. 181)
2. “Prosecutors duty to disclose exculpatory evidence, does not however apply to grand jury
proceedings. It is a separate constitutional body not subject to plenary judicial supervision.”
3. Brady Rule does not provide to grand jury proceedings (grand jury not part of judicial process)
4. P only need disclose to D the weaknesses in IDs
iii. Is there a duty post indictment?
1. Yes, R 3.8(d), duty to disclose information that tends to negate guilt or mitigate the offense
iv. What if prosecutor doesn’t disclose and defendant latter finds out?
1. Nothing, it’s a professional responsibility rule
v. Brady v. Maryland (Constitutional Duty)
1. The suppression by the prosecution of evidence favorable to the accused violates due process
where the evidence is material to either guilt or punishment
2. Higher standard than the ethical rule
vi. Ethical Duty (R 3.8(d))
1. Requires disclosure of all information that tends to negate guilt or mitigate the offense
vii. Prosecutor should use the “tends to” standard
viii. Materiality is the standard by which a court determines if a defendant has been harmed by a nondisclosure
ix. How should the defense frame a request for exculpatory evidence
1. Using R 3.8(d)
23
g. United States v. Talao (p. 185)
i. I- Has the prosecutor violated R 4.2, breaching the attorney-client privilege? Ex parte communication with
a party represented by counsel
1. NO, falls within the R 4.2 exception allowed by law
ii. R 4.2 Comment 5
1. Authorized by law
2. Communications between a target and prosecutor are authorized by law if they do not violate the
defendant’s 6th Amendment right to counsel (traditional)
3. Modern view – Court should examine all facts and circumstances to determine if a pre-indictment
communication is authorized by law
a. Factors:
i. Has an adversarial posture developed
ii. Who initiated the contact
iii. Is perjury or obstruction involved
iv. Does the multiple representation create an actual conflict
v. Did government advise the witness of a right to independent counsel
vi. Has the government engaged in misconduct
4. McDade Act
a. A state supreme court interpretation of R.4.2 will be binding on both state and federal
prosecutors.
h. Problem 2-10(b)
i. Can we talk to Norwood?
ii. R 4.2 Comment 5
1. Is communication authorized by law? Talao factors
a. Probably it is
iii. Is it permissible to file charges against the mother if the father and son don’t take the plea
1. Vindictiveness
a. Objective evidence that the prosecutor acted to punish the defendant for standing on his
legal rights (punishing the defendant for something he has a right to do)
b. Does not apply to plea bargaining, because they are give and take negotiations; therefore,
it’s acceptable
c. NOTE: vindictiveness is usually not the right answer on the bar exam
i. Problem 2-11 Fee Forfeiture and Lawyer Subpoenas
i. Money Laundering
1. Knowingly engage or attempt to engage in a monetary transaction in criminally derived property
that is of a value greater than $10,000 and is derived from specified unlawful activity
ii. Civil Forfeiture
1. Money derived from illegal activity subject to forfeiture, lawyer must obtain verification that
money is not tainted
iii. Tax Reporting
1. Receipt of cash [a cashier’s check but not a personal check] of $10,000 or more must be reported
to the IRS
iv. Lawyer has duty to know where all funds come from and a duty to investigate the source of all funds
v. Who’s in charge?
1. R 1.0(e) – Informed Consent
a. Insures client autonomy by providing the client with all the information necessary to
understand the benefits and risks associated with the decision she must make.
i. Based on the client’s ability to comprehend
2. R 2.1 – Lawyer as counselor
a. Independent professional judgment
b. Should not be guided by personal considerations
c. Agency principles
i. Actual authority – principal has provided agent w/ power to make (certain)
decisions in her interest
ii. Apparent authority – third person believes agent has authority to bind principal
when in fact agent does not. If principal contributed to 3rd parties’ belief then
principal may be bound by estoppel
iii. Law practice – 3rd party (other side’s atty) may reasonably believe that atty has
authority over strategic

24
3. R 1.4 – Client Communications
a. Largest source of grievances
4. R 1.14 – Dealing with clients with diminished capacity (paternalism)
5. R 1.2 (paternalism)

Rule 1.0: Terminology


(o.) “Belief” or “believes” denotes that the person involved actually supposed the fact in question to be true. A person’s belief may be inferred from
circumstances.
(p.) “Confirmed in writing,” when used in reference to the informed consent of a person, denotes informed consent that is given in writing by the person or a
writing that a lawyer promptly transmits to the person confirming an oral informed consent. See paragraph (e) for the definition of “informed consent.” If it
is not feasible to obtain or transmit the writing at the time the person gives informed consent, then the lawyer must obtain or transmit it within a reasonable
time thereafter.
(q.) “Firm” or “law firm” denotes a lawyer or lawyers in a law partnership, professional corporation, sole proprietorship or other association authorized to
practice law; or lawyers employed in a legal services organization or the legal department of a corporation.
(r.) “Fraud” or “fraudulent” denotes conduct that is fraudulent under the substantive or procedural law of the applicable jurisdiction and has a purpose to
deceive.
(s.) “Informed consent” denotes the agreement by a person to a proposed course of conduct after the lawyer has communicated adequate information and
explanation about the material risks of and reasonably available alternatives to the proposed course of conduct.
(t.) “Knowingly,” “know”, or “knows” denotes actual knowledge of the fact in question. A person’s knowledge may be inferred from circumstances.
(u.) “Partner” denotes a member of a partnership, a shareholder in a law firm organized as a professional corporation, or a member of an association authorized
to practice law.
(v.) “Reasonable” or “reasonably” when used in relation to conduct by a lawyer denotes the conduct of a reasonably prudent and competent lawyer.
(w.) “Reasonable belief” or “reasonably believes” when used in reference to a lawyer denotes that the lawyer believes the matter in question and that the
circumstances are such that the belief is reasonable.
(x.) “Reasonably should know” when used in reference to a lawyer denotes that a lawyer of reasonable prudence and competence would ascertain the matter in
question.
(y.) “Screened” denotes the isolation of a lawyer from any participation in a matter through the timely imposition of procedures within a firm that are reasonably
adequate under the circumstances to protect information that the isolated lawyer is obligated to protect under these Rules or other laws.
(z.) “Substantial” when used in reference to degree or extent denotes a material matter of clear and weighty importance.
(aa.) “Tribunal” denotes a court, an arbitrator in a binding arbitration proceeding or a legislative body, administrative agency or other body acting in an
adjudicative capacity. A legislative body, administrative agency or other body acts in an adjudicative capacity when a neutral official, after the presentation
of evidence or legal argument by a party or parties, will render a binding legal judgment directly affecting a party’s interest in a particular matter.
(bb.) “Writing” or “written” denotes a tangible or electronic record of a communication or representation, including handwriting, typewriting, printing,
photostating, photography, audio or video recording and e-mail. A “signed” writing includes an electronic sound, symbol or process attached to or logically
associated with a writing and executed or adopted by a person with the intent to sign the writing.

Rule 1.2: Scope of Representation and Allocation of Authority Between Client and Lawyer
(a.) Subject to paragraphs (c) and (d), a lawyer shall abide by a client’s decisions concerning the objectives of representation and, as required by Rule 1.4, shall consult
with the client as to the means by which they are to be pursued. A lawyer may take such action on behalf of the client as is impliedly authorized to carry out the
representation. A lawyer shall abide by a client’s decision whether to settle a matter. In a criminal case, the lawyer shall abide by the client’s decision, after consultation
with the lawyer, as to a plea to be entered, whether to waive jury trial and whether the client will testify.
(b.) A lawyer’s representation of a client, including representation by appointment, does not constitute an endorsement of the client’s political, economic, social or
moral views or activities.
(c.) A lawyer may limit the scope of the representation if the limitation is reasonable under the circumstances and the client gives informed consent.
(d.) A lawyer shall not counsel a client to engage, or assist a client, in conduct that the lawyer knows is criminal or fraudulent, but a lawyer may discuss the legal
consequences of any proposed course of conduct with a client and may counsel or assist a client to make a good faith effort to determine the validity, scope, meaning or
application of the law.

Rule 1.4: Communication


(a.) A lawyer shall:
(1.) promptly inform the client of any decision or circumstance with respect to which the client’s informed consent, as
defined by Rule 1.0(e), is required by these Rules;
(2.) reasonably consult with the client about the means by which the client’s objectives are to be accomplished;
(3.) keep the client reasonably informed about the status of the matter;
(4.) promptly comply with reasonable request for information; and
(5.) consult with the client about any relevant limitation on the lawyer’s conduct when the lawyer knows that the client
expects assistance not permitted by the Rules of Professional Conduct or other law.
(b.) A lawyer shall explain a matter to the extent reasonably necessary to permit the client to make informed decisions regarding the representation.

Rule 1.5: Fees


(a.) A lawyer shall not make an agreement for, charge, or collect an unreasonable fee or an unreasonable amount for expenses. The factors to be considered in
determining the reasonableness of the fee include the following:
(1.) the time and labor required, the novelty and difficulty or the questions involved, and the skill requisite to perform the
legal service properly
(2.) the likelihood, if apparent to the client, that the acceptance of the particular employment will preclude other
employment by the lawyer;
(3.) the fee customarily charged in the locality for similar legal services;
(4.) the amount involved and the results obtained;
(5.) the time limitations imposed by the client or by the circumstances;
(6.) the nature and length of the professional relationship with the client;
(7.) the experience, reputation, and ability of the lawyer or lawyers performing the services; and
(8.) whether the fee is fixed or contingent

25
(b.) The scope of the representation and the basis or rate of the fee and expenses for which the client will be responsible shall be communicated to the client, preferably
in writing, before or within a reasonable time after commencing the representation, except when the lawyer will charge a regularly represented client on the same basis
or rate. Any changes in the basis or rate of the fee or expenses shall also be communicated to the client.
(c.) A fee may be contingent on the outcome of the matter for which the service is rendered, except in a matter in which a contingent fee is prohibited by paragraph (d)
or other law. A contingent fee agreement shall be in a writing signed by the client and shall state the method by which the fee is to be determined, including the
percentage or percentages that shall accrue to the lawyer in the event of settlement, trial or appeal; litigation and other expenses to be deducted from the recovery; and
whether such expenses are to be deducted before or after the contingent fee is calculated. The agreement must clearly notify the client of any expenses for which the
client will be liable whether or not the client is the prevailing party. Upon conclusion of a contingent fee matter, the lawyer shall provide the client with a written
statement stating the outcome of the matter and, if there is a recovery, showing the remittance to the client and the method of its determination.
(d.) A lawyer shall not enter into an arrangement for, charge, or collect:
(1.) any fee in a domestic relations matter, the payment or amount of which is contingent upon the securing of a divorce or
upon the amount of alimony or support, or property settlement in lieu thereof; or
(2.) a contingent fee for representing a defendant in a criminal case
(e.) A division of a fee between lawyers who are not in the same firm may be made only if:
(1.) the division is in proportion to the services performed by each lawyer or each lawyer assumes joint responsibility for the representation;
(2) the client agrees to the arrangement, including the share each lawyer will receive, and the agreement is confirmed in
writing; and
(3.) the total fee is reasonable.

Rule 1.6 Confidentiality of Information


(a)A lawyer shall not reveal information relating to the representation of a client unless the client gives informed consent, the disclosure is impliedly authorized in order
to carry out the representation or the disclosure is permitted by paragraph (b).
(b.)A lawyer may reveal information relating to the representation of a client to the extent the lawyer reasonably believes necessary:
(1.) to prevent reasonably certain death or substantial bodily harm;
(2.) to prevent the client from committing a crime or fraud that is reasonably
certain to result in substantial injury to the financial interest or property of another and in furtherance of which the client has used or is using the lawyer’s
services.
(3.) to prevent, mitigate or rectify substantial injury to the financial interest or property of another that is reasonably certain to result or has resulted from the
client’s commission of a crime or fraud in furtherance of which the client has used the lawyer’s services;
(4.) to secure legal advice about the lawyer’s compliance with these Rules;
(5.) to establish a claim or defense on behalf of the lawyer in a controversy between the lawyer and the client, to establish a defense to a criminal charge or
civil claim against the lawyer based upon conduct in which the client was involved, or to respond to allegations in any proceeding concerning the lawyer’s
representation of the client; or
(6.) to comply with other law or a court order

Rule 1.7: Conflict of Interest: Current Clients


(a.) Except as provided in paragraph (b), a lawyer shall not represent a client if the representation involves a concurrent conflict of interest. A concurrent conflict of
interest exists if:
(1.) the representation of one client will be directly adverse to another client; or
(2.) there is a significant risk that the representation of one or more clients will be materially limited by the lawyer’s
responsibilities to another client, a former client or a third person or by a personal interest of the lawyer.
(b.) Notwithstanding the existence of a concurrent conflict of interest under paragraph (a), a lawyer may represent a client if:
(1.) the lawyer reasonably believes that the lawyer will be able to provide competent and diligent representation to each
affected client;
(2.) the representation is not prohibited by law;
(3.) the representation does not involve the assertion of a claim by one client against another client represented by the
lawyer in the same litigation or other proceeding before a tribunal; and
(4.) each affected client gives informed consent, confirmed in writing.

Rule 1.8: Conflict of Interest: Current Clients: Specific Rules


(a.) A lawyer shall not enter into a business transaction with a client or knowingly acquire an ownership, possessory, security or other pecuniary interest adverse to a
client unless:
(1.) the transaction and terms on which the lawyer acquires the interest are fair and reasonable to the client and are fully
disclosed and transmitted in writing in a manner that can be reasonably understood by the client;
(2.) the client is advised in writing of the desirability of seeking and is given a reasonable opportunity to seek the advice of
independent legal counsel on the transaction; and
(3.) the client gives informed consent, in a writing signed by the client, to the essential terms of the transaction, including
whether the lawyer is representing the client in the transaction.
(b.) A lawyer shall not use information relating to representation of a client to the disadvantage of the client unless the client gives informed consent, except as
permitted or required by these Rules.
(c.) A lawyer shall not solicit any substantial gift from a client, including a testamentary gift, or prepare on behalf of a client an instrument giving the lawyer or a person
related to the lawyer any substantial gift unless the lawyer or other recipient of the gift is related to the client. For purposes of this paragraph, related persons include a
spouse, child, grandchild, parent, grandparent, or other relative or individual with whom the lawyer or the client maintains a close, familial relationship.
(d.) Prior to the conclusion of representation of a client, a lawyer shall not make or negotiate an agreement giving the lawyer literary or media rights to a portrayal or
account based in substantial part on information relating to the representation.
(e.) A lawyer shall not provide financial assistance to a client in connection with the pending or contemplated litigation, except that:
(1.) a lawyer may advance court costs and expenses of litigation, the repayment of which may be contingent on the
outcome of the matter; and
(2.) a lawyer representing an indigent client may pay court costs and expenses on behalf of the client.
(f.) A lawyer shall not accept compensation for representing a client for one other than the client unless:
(1.) the client gives informed consent
(2.) there is no interference with the lawyer’s independence of professional judgment or with the client-lawyer relationship;
and
(3.) information relating to representation of a client is protected as required by Rule 1.6

26
(g.) A lawyer who represents two or more clients shall not participate in making an aggregate settlement of the claims of or against the clients, or in a criminal case an
aggregated agreement as to guilty or nolo contendere pleas, unless each client gives informed consent, in a writing signed by the client. The lawyer’s disclosure shall
include the existence and nature of all the claims or pleas involved and of the participation of each person in the settlement
(h.) A lawyer shall not:
(1.) make an agreement prospectively limiting the lawyer’s liability to a client for malpractice unless the client is
independently responsible in making the agreement; or
(2.) Settle a claim or potential claim for such liability with an unrepresented client or former client unless that person is
advised in writing of the desirability of seeking and is given a reasonable opportunity to seek the advice of independent legal counsel in connection
therewith.
(i.) A lawyer shall not acquire a proprietary interest in the cause of action or subject matter of litigation the lawyer is conducting for a client, except that a lawyer may:
(1.) acquire a lien authorized by law to secure the lawyer’s fee or expenses; and
(2.) contract with a client for a reasonable contingent fee in a civil case.
(j.) A lawyer shall not have sexual relations with a client unless, a consensual sexual relationship existed between them when the client-lawyer relationship commenced
(k.) While lawyers are associated in a firm, a prohibition in the foregoing paragraphs (a) through (i) that applies to any one of them, shall apply to all of them.

Rule 1.9: Duties to Former Clients


(a.) A lawyer who has formerly represented a client in a matter shall not thereafter represent another person in the same or a substantially related matter in which that
person’s interest are materially adverse to the interest of the former client unless the former client gives informed consent, confirmed in writing.
(b.) A lawyer shall not knowingly represent a person in the same or a substantially related matter in which a firm with which the lawyer formerly was associated had
previously represented a client
(1.) whose interest are materially adverse to that person; and
(2.) about whom the lawyer had acquired information protected by Rule 1.6 and 1.9(c) that is material to the matter;
unless the former client gives informed consent, confirmed in writing.
(c.) A lawyer who has formerly represented a client in a matter or whose present or former firm has formerly represented a client in a matter shall not thereafter:
(1.) use information relating to the representation to the disadvantage of the former client except as these Rules would
permit or require with respect to a client, or when the information has become generally known; or
(2.) reveal information relating to the representation except as these Rules would permit with respect to a client.

Rule 1.10 Imputation of Conflicts of Interest: General Rule


(a.) While lawyers are associated in a firm, none of them shall knowingly represent a client when any one of them practicing alone would be prohibited from doing so
by Rules 1.7 or 1.9, unless the prohibition is based on a personal interest of the prohibited lawyer and does not present a significant risk of materially limiting the
representation of the client by the remaining lawyers in the firm.
(b.) When a lawyer has terminated an association with a firm, the firm is not prohibited from thereafter representing a person with interest materially adverse to those of
a client represented by the firm, unless:
(1.) the matter is the same or substantially related to that in which the formerly associated lawyer represented the client;
and
(2.) any lawyer remaining in the firm has information protected by Rules 1.6 and 1.9(c) that is material to the matter.
(c.) A disqualification prescribed by this rule may be waived by the affected client under the conditions stated in Rule 1.7.
(d.) The disqualification of lawyers associated in a firm with former or current government lawyers is governed by Rule 1.11

Rule 1.13: Organization as Client


(a.) A lawyer employed or retained by an organization represents the organization acting through its duly authorized constituents.
(b.) If a lawyer for an organization knows that an officer, employee or other person associated with the organization is engaged in action, intends to act or refuses to act
in a matter related to the representation that is a violation of a legal obligation to the organization, or a violation of a law that reasonably might be imputed to the
organization, and that is likely to result in substantial injury to the organization, then the lawyer shall proceed as is reasonably necessary in the best interest of the
organization. Unless the lawyer reasonably believes that it is not necessary in the best interest of the organization to do so, the lawyer shall refer the matter to higher
authority in the organization, including, if warranted by the circumstances, to the highest authority that can act on behalf of the organization as determined by applicable
law.
(c.) Except as provided in paragraph (d), if
(1.) despite the lawyer’s efforts in accordance with paragraph (b) the highest authority that can act on behalf of the
organization insist upon or fails to address in a timely and appropriate manner an action or a refusal to act, that is clearly a violation of law, and
(2.) the lawyer reasonably believes that the violation is reasonably certain to result in substantial injury to the organization,
then the lawyer may reveal information relating to the representation whether or not Rule 1.6 permits such disclosure, but only if and to the extent the lawyer reasonably
believes necessary to prevent substantial injury to the organization
(d.) Paragraph (c) shall not apply with respect to information relating to a lawyer’s representation of an organization to investigate an alleged violation of law, or to
defend the organization or an officer, employee or other constituent associated with the organization against a claim arising out of an alleged violation of law.
(e.) A lawyer who reasonably believes that he or she has been discharged because of the lawyer’s actions taken pursuant to paragraphs (b) and (c), or who withdraws
under circumstances that require or permit the lawyer to take action under either of those paragraphs, shall proceed as the lawyer reasonably believes necessary to
assure that the organization’s highest authority is informed of the lawyer’s discharge or withdrawal.
(f.) In dealing with an organization’s directors, officers, employees, members, shareholders or other constituents, a lawyer shall explain the identity of the client when
the lawyer knows or reasonably should know that the organization’s interest are adverse to those of the constituents with whom the lawyer is dealing.
(g.) A lawyer representing an organization may also represent any of its directors, officers, employees, members, shareholders or other constituents, subject to the
provisions of Rule 1.7. If the organization’s consent to the dual representation is required by Rule 1.7, the consent shall be given by an appropriate official of the
organization other than the individual who is to be represented, or by the shareholders.

Rule 1.14: Client with Diminished Capacity


(a.) When a client’s capacity to make adequately considered decisions in connection with a representation is diminished, whether because of minority, mental
impairments or for some other reason, the lawyer shall, as far as reasonably possible, maintain a normal client-lawyer relationship with the client.
(b.) When the lawyer reasonably believes that the client has diminished capacity, is at risk of substantial physical, financial or other harm unless action is taken and
cannot adequately act in the client’s own interest, the lawyer may take reasonably necessary protective action, including consulting with individuals or entities that have
the ability to take action to protect the client and, in appropriate cases, seeking the appointment of a guardian ad litem, conservator or guardian.
(c.) Information relating to the representation of a client with diminished capacity is protected by Rule 1.6. When taking protective action pursuant to paragraph (b), the
lawyer is impliedly authorized under Rule 1.6(a) to reveal information about the client, but only to the extent reasonably necessary to protect the client’s interest.

Rule 1.15: Safekeeping Property


(a) A lawyer shall hold property of clients or third persons that is in the lawyer’s possession in connection with a representation separate from the lawyer’s own

27
property. Funds shall be kept in a separate account maintained in the state where the lawyer’s office is situated, or elsewhere with the consent of the client or third
person. Other property shall be identified as such and appropriately safeguarded. Complete records of such account funds and other property shall be kept by the lawyer
and shall be preserved for a period of five years after termination of representation.
(b.) A lawyer may deposit the lawyer’s own funds in a client trust account for the sole purpose of paying bank service charges on that account, but only in an amount
necessary for that purpose.
(c.) A lawyer shall deposit into a client trust account legal fees and expenses that have been paid in advance, to be withdrawn by the lawyer only as fees are earned or
expenses incurred.
(d.) Upon receiving funds or other property in which a client or third person has an interest, a lawyer shall promptly notify the client or third person. Except as stated in
this rule or otherwise permitted by law or by agreement with the client, a lawyer shall promptly deliver to the client or third person any funds or other property that the
client or third person is entitled to receive and, upon request by the client or third person, shall promptly render a full accounting regarding such property.
(e.) When in the course of representation a lawyer is in possession of property in which two or more persons (one of whom may be the lawyer) claim interests, the
property shall be kept separate by the lawyer until the dispute is resolved. The lawyer shall promptly distribute all portions of the property as to which the interests are
not in dispute.

Rule 1.16: Declining or Terminating Representation


(a.) Except as stated in paragraph (c), a lawyer shall not represent a client or, where representation has commenced, shall withdraw from the representation of a client if:
(1.) the representation will result in a violation of the rules of professional conduct or other law;
(2.) the lawyer’s physical or mental condition materially impairs the lawyer’s ability to represent the client; or
(3.) the lawyer is discharged
(b.) Except as stated in paragraph (c), a lawyer may withdraw from representing a client if:
(1.) withdrawal can be accomplished without material adverse effect on the interest of the client;
(2.) the client persists in a course of action involving the lawyer’s services that the lawyer reasonably believes is criminal or fraudulent;
(3.) the client has used the lawyer’s services to perpetrate a crime or fraud;
(4.) the client insists upon taking action that the lawyer considers repugnant or with which the lawyer has a fundamental disagreement;
(5.) the client fails substantially to fulfill an obligation to the lawyer regarding the lawyer’s services and has been given reasonable warning that the lawyer
will withdraw unless the obligation is fulfilled;
(6.) the representation will result in an unreasonable financial burden on the lawyer or has been rendered unreasonably difficult by the client; or
(7.) other good cause for withdrawal exists
(c.) A lawyer must comply with applicable law requiring notice to or permission of a tribunal when terminating a representation. When ordered to do so by a tribunal, a
lawyer shall continue representation notwithstanding good cause for terminating the representation.
(d.) Upon termination of representation, a lawyer shall take steps to the extent reasonably practicable to protect a client’s interests, employment of other counsel,
surrendering papers and property to which the client is entitled and refunding any advance payment of fee or expense that has not been earned or incurred. The lawyer
may retain papers relating to the client to the extent permitted by other law.

Rule 2.1: Advisor


In representing a client a lawyer shall exercise independent professional judgment and render candid advice. In rendering advice, a lawyer may refer not only to law but
to other considerations such as moral, economic, social and political factors, that may be relevant to the client’s situation.

Rule 3.1: Meritorious Claims and Contentions


A lawyer shall not bring or defend a proceeding, or assert or controvert an issue therein, unless there is a basis in law and fact for doing so that is not frivolous, which
includes a good faith argument for an extension, modification or reversal of existing law. A lawyer for the defendant in a criminal proceeding, or the respondent in a
proceeding that could result in incarceration, may nevertheless so defend the proceeding as to require that every element of the case be established.

Rule 3.2; Expediting Litigation


A lawyer shall make reasonable efforts to expedite litigation consistent with the interest of the client.

Rule 3.4: Fairness to Opposing Party and Counsel


A lawyer shall not:
(a.) unlawfully obstruct another party’s access to evidence or unlawfully alter, destroy or conceal a document or other material having potential evidentiary value. A
lawyer shall not counsel or assist another person to do any such act;
(b.) falsify evidence, counsel or assist a witness to testify falsely, or offer an inducement to a witness that is prohibited by law;
(c.) knowingly disobey an obligation under the rules of a tribunal, except for an open refusal based on an assertion that no valid obligation exists;
(d.) in pretrial procedure, make a frivolous discovery request or fail to make reasonably diligent effort to comply with a legally proper discovery request by an opposing
party.
(e.) in trial, allude to any matter that the lawyer does not reasonably believe is relevant or that will not be supported by admissible evidence, assert personal knowledge
of facts in issue except when testifying as a witness, or state a personal opinion as to the justness of a cause, the credibility of a witness, the culpability of a civil litigant
or the guilt or innocence of an accused; or
(f.) request a person other than a client to refrain from voluntarily giving relevant information to another party unless:
(1.) the person is a relative or an employee or other agent of a client; and
(2.) the lawyer reasonably believes that the person’s interest will not be adversely affected by refraining from giving such
information.

Rule 3.7: Lawyer as Witness


(a.) A lawyer shall not act as advocate at a trial in which the lawyer is likely to be a necessary witness unless:
(1.) the testimony relates to an uncontested issue;
(2.) the testimony related to the nature and value of legal services rendered in the case; or
(3.) disqualification of the lawyer would work substantial hardship on the client.
(b.) A lawyer may act as advocate in a trial in which another lawyer in the lawyer’s firm is likely to be called as a witness unless precluded from doing so by Rule 1.7 or
Rule 1.9.

Rule 4.2: Communication with Person Represented by Counsel


In representing a client, a lawyer shall not communicate about the subject of the representation with a person the lawyer knows to be represented by another lawyer in
the matter, unless the lawyer has the consent of the other lawyer or is authorized to do so by law or a court order.

Rule 4.3: Dealing With Unrepresented Person

28
In dealing on behalf of a client with a person who is not represented by counsel, a lawyer shall not state or imply that the lawyer is disinterested. When the lawyer
knows or reasonably should know that the unrepresented person misunderstands the lawyer’s role in the matter, the lawyer shall make reasonable efforts to correct the
misunderstanding. The lawyer shall not give legal advice to an unrepresented person, other than the advice to secure counsel, if the lawyer knows or reasonably should
know that the interest of such a person are or have a reasonable possibility of being in conflict with the interest of the client.

Chapter 3: Ethical Issus in Civil Litigation: The Client-Lawyer Relationship, Confidentiality, and Conflicts of Interest
I. The Client-Lawyer Relationship
a. Problem 3-1: Contingent Fees, Expenses, and Fee Splitting
i. I- Was the contingent fee reasonable?
1. R 1.5
a. Look at the local practice
i. Some jurisdictions bars/courts have established
b. R 1.5(a)
i. Time, labor required, novelty, custom, results, time limitations, preclusion of
other work
2. Attorney can vary fees or charge a flat rate or a percentage depending on where in the stage of
litigation they are
3. Expert’s fee cannot be a contingent fee (R 3.4 Comment 3)
4. Fact witnesses can have their expenses reimbursed
5. General overhead can only be charged to the client with their consent
6. R 1.5(c)
a. Lawyer must clearly notify the client for any expenses for which the client will be liable
b. Must specify whether expenses are to be deducted before or after the contingent fee is
calculated
c. Upon conclusion of a contingent fee lawyer must provide with a settlement statement
7. R 1.5(e) – Fee Divisions must be:
a. Reasonableness
i. In proportion to services performed, or
ii. Pursuant to joint representation
b. And, with client consent
8. Client’s recourse in fee disputes
a. Grievance
b. Suit for damages
c. Lawyer’s duty to report misconduct
9. HYPO:
a. While Harriet’s case was awaiting trial her house was struck by lighting and burned
down. Her lawyer advanced her enough money to find an apartment and to pay her rent
until she received a settlement from her homeowner’s insurance. They both agreed that
the funds would be repaid from a settlement or judgment on the malpractice action.
i. Not permissible R 1.8 (e) – Conflict – Personal interest in the claim
b. Problem 3-2 Engagement and Nonengagement Agreements:
i. Can a firm create malpractice liability for itself by turning down a case?
1. Yes, the attorney must advise them to seek other counsel and advise statute of limitations
2. Togstad v. Vesely, Otto, Miller & Keefe (p.231)
a. Elements of malpractice:
i. Established attorney-client relationship
ii. Attorney negligence or BoK
iii. Negligence was the proximate cause of plaintiff’s damages
iv. But for the defendant’s conduct plaintiff would have prevailed
b. Liable in this case
i. Didn’t look into the claim or consult an expert
ii. Gave client advice
iii. Did not provide client with means to protect their interest
c. Way to avoid liability
i. Nonegagement Letter
1. Purpose:
a. To let client know we do not have a lawyer-client relationship
b. Client should seek another lawyer ASAP
2. When does an attorney-client relationship exist?
a. When the client believes one exist
29
i. The point of the nonengagement letter is to make this
unreasonable
3. Must include:
a. Declination of representation
b. Seek advice of other attorney
c. Advise of any procedural deadlines
4. Protect:
a. Lawyer (Defensive)
b. Client (Educational)
5. Letters should not provide reasons why the attorney declines the
representation
d. R 1.18 Duties to Prospective Clients
i. Confidentiality
ii. Protection of client’s information from use by another client of the firm in a
substantially related matter
ii. Engagement Letters
1. Purpose
a. Educate client -- Educational
b. Protect the attorney -- Defensive
c. Protect Client’s claim
2. How do you present it to the client?
a. It’s a contract, should be negotiated
b. Cannot be presented as a take it or leave it order
c. Go through it point by point with the client to make sure they agree
3. How do rules construct a client?
a. As able to understand and negotiate a contract
b. In the state of mind that permits bargaining
4. Form:
a. Varies depending on the client and firm
b. Requirements:
i. Must state the parties
ii. Contain any consents needed or required (i.e. multiple representation and
waivable conflicts)
iii. Scope of representation
1. Goals of client
2. How long the representation will last
iv. Firm representation
1. The client is represented by the firm not a particular attorney
v. Efforts
1. Both the attorney and client will use their best efforts
vi. Fee Arrangement
vii. Billing
viii. Appeals
ix. Termination
5. Tone:
a. Depends on the client (friendly v. formal)
6. Process
a. Recognizes mutuality and bargaining
7. Informed Consent
a. Achieved when the client can explain the engagement agreement in their own words
c. Problem 3-3 Confidentiality
i. P 3-3(a) Communication types
1. R 1.6 Confidentiality
a. Security of various forms of communication
b. STANDARD:
i. Whether the technology provides a reasonable expectation of privacy
c. What policies should be adopted
i. Develop based on potential consequences
ii. Must comply with duty of care
1. Negligence standard

30
iii. Should be included in the engagement agreement
d. ECPA
i. Makes interception of communications a crime and inadmissible in court
ii. P 3-3(b)
1. Are the documents subject to attorney-client privilege
a. Two exceptions to the attorney-client privilege
i. Waiver
ii. Crime fraud exception
b. The privilege belongs to the client
i. Can be waived inadvertently by client or counsel
1. Counsel would be liable to client
ii. Can be waived through:
1. Agreement
2. Disclosure by the plaintiff to a third party
3. Failure to object
4. Disclaimer
5. In suits against the attorney or based on ineffective assistance of
counsel
2. Attorney Work Product
a. To qualify:
i. The material records or reflects litigation investigation or analysis
ii. The material was prepared by or for a party or a party’s representative, including
a party’s lawyer, consultant, surety, indemnitor, insurer or agent
iii. The material was prepared in anticipation of litigation, i.e., litigation in progress
or prospective litigation
1. Although almost all of the work which attorneys do and the advice
which they dispense may be viewed as being in anticipation of
litigation or its avoidance, work product immunity requires a more
immediate showing than the remote possibility of litigation. There must
be a substantial probability that litigation will occur and that
commencement of such litigation is imminent, or at least litigation must
reasonably be anticipated … Thus, advising a client about matters
which may ultimately come to litigation is insufficient for such advice
to be considered to have been rendered in anticipation of litigation; the
threat of litigation must be more real and imminent than that. The test
is not whether litigation actually has begun, however. [AMJUR
Depositions sec. 47]
iv. Ordinary Work Product v. Opinion Work Product
1. Ordinary: Standard
a. Substantial need and inability to get information absent undue
hardship
2. Opinion: Standard
a. Extraordinary Circumstances
b. Crime Fraud Exception applies
3. Who is the client? – Corporate Client
a. Upjohn Test, Subject Matter, Control Group
b. R 4.2 COMMENT 7
i. The Client is someone who Supervises, directs or consults with organization’s
lawyer, or
ii. Is an agent of the organization such that:
1. Her or his statements will bind the organization, or
2. Her or his acts or omissions will be attributed to the organization
4. Leroy Lawyer is a mechanical engineer and a member of the bar of Model State. He chairs the
design and drafting department at Model State Motors, Inc. and regularly has discussions with the
company’s general counsel. Following a series of accidents involving Model State Motors
automobiles, Plaintiffs sue and move to compel production of technical drawings prepared by
Leroy concerning the safety of Model State Motors’ automobiles. The drawing appearing below
was prepared three years before Plaintiff’s accident for a talk Leroy gave for senior Model State

31
Motors staff entitled, “Designing Better Cars.” It has been identified as being responsive to the
production request.
a. The document is not privileged because Leroy was not acting as a lawyer when he
drafted it.
i. Anticipation of litigation
b. The document is not privileged because it is a drawing.
c. The document is not privileged because it neither provides nor requests legal advice.
i. CORRECT, no legal advice, document doesn’t say anything
d. The document is not privileged because Leroy is not the client.

iii. See Question above. After Plaintiff’s accident the president of Model State Motors asked Leroy to prepare
the document appearing below. Will Model State Motors have to produce this document to the Plaintiffs?
1. The document will not have to be produced because the document was prepared in
anticipation of litigation.
a. CORRECT, after the accident it was done = anticipation of litigation
2. The document will not have to be produced because the document is privileged.
a. No indication that it was going to legal counsel
3. The document will have to be produced because Leroy is not the client.
a. Might be
4. The document will have to be produced because it is not confidential.
a. Don’t know that

32
iv. See Questions above. Plaintiff’s lawyers wish to informally interview Leroy about the frequent
conversations he has had with Model State Motor’s general counsel over the last two years.
1. Leroy is the client so he can consent to be informally interviewed by Plaintiff’s attorney.
2. Leroy may be informally interviewed by Plaintiff’s attorney because he is not the client.
3. Leroy may not be informally interviewed by Plaintiff’s attorney because Leroy is an
attorney.
4. Leroy may not be informally interviewed by Plaintiff’s attorney unless Model State Motor’s
attorney consents.
a. CORRECT R 4.2
II. Conflicts:
a. R 1.7
i. Concurrent Conflicts:
1. Direct Adversity, or
2. Material Limitation
ii. Consentable Conflicts
1. Attorney’s reasonable belief he can provide competent and diligent representation, and
2. All parties consent
iii. Nonconsentable Conflicts
1. Prohibited by law [buyer & seller; multiple criminal defendants; clients w/o ability to provide
informed consent, e.g., municipalities Comment 16]
2. Directly adverse claims
b. Types of Conflicts
i. Adverse Representation against a Current Clients
ii. Multiple Representation of clients in a single matter
iii. Representation against a Former Clients
iv. Advocate - Witness Conflict
v. Client vs. Lawyer’s personal interests
c. Problem 3-4
i. R 1.7 Comment 2: Current Client v. Former Client
1. Grievance Committee v. Rottner: (p. 264)
a. Fiduciary relationship requires a duty of loyalty from attorney to the client
b. “When a client engages the services of a lawyer in a given piece of business he is entitled
to feel that, until that business is finally disposed of in some manner, he has the undivided
loyalty of the one upon whom he looks as his advocate and his champion.”
2. Direct adversity would exist if both current clients, if not R 1.7(b) may allow with informed
consent
3. Who is a client? R 1.13 (Entity)

33
a. (a) – Organization is client
i. Attorney represents the entity not its components
1. Compatible with Rule 4.2 Comment 7?
b. (b) – Duty to inquire/report
i. Attorney must act when constituents are not working in organizations best
interest or violating the law
c. (c) – Discretion to disclose
i. Exception to R 1.6 if the activity is illegal or fraudulent and there is a reasonable
certainty of substantial injury
d. (d) – Investigation exception
i. (d) limits (c), if you are hired to investigate wrongdoing, you aren’t under
obligation to report that
e. (e) – Termination exception
i. Exception to R 1.16
ii. If fired or withdraw because of action still have discretion to alert the highest
authority
f. (f) – Conflicts
i. Attorney must explain he represent the corporation when he knows or
reasonable should know the constituents interest is adverse
g. (g) – Multiple Representation
i. Can represent an individual within the organization subject to R 1.7
4. What is VS paid a general retainer?
a. He acquired your services, it’s a payment for availability (current client), can’t represent
GC
5. If it is unclear and want to take on representation
a. Run through R1.7 Analysis
i. Significant risk of material limitation
ii. If yes, does the attorney have a reasonable belief
iii. Can he get informed consent from each
6. If VS is a former client what are your responsibilities?
a. R 1.9 does not require informing former clients if matters are not substantially related
7. If there is a small matter with VS can the firm withdrawal from VS to accept GC in a more
lucrative matter?
a. R 1.16(b)(1)
i. Can withdrawal be accomplished without a material adverse effect on the client?
b. “Hot potato”
i. If long history of representation shouldn’t drop them
ii. If purely financial shouldn’t
8. Prospective waivers of conflicts
a. Usually good only with sophisticated clients
b. Otherwise probably not enforceable
c. Elements of:
i. Sophisticated client
ii. Are they represented by independent counsel or had the opportunity to be
iii. Informed consent
iv. Specialized representation
1. If elements are not present  probably not enforceable
d. Problem 3-4 (b)
i. R 1.13 Representing a partnership
1. I: Who is the client?
a. The partnership is the client, not the individual partners
2. What might establish a relationship with an individual partner
a. Has he met with the individual member and provided advice
b. Is the member independently represented
c. Has the member personally paid fees
i. Avoid by clearly stating who you represent in your engagement letter
3. Firm could represent both if R 1.7(b) followed, but risky
e. Problem 3.4(c)

34
i. Is it permissible to represent Clawson against former employer when the former employer is a member of
an association that K&M represents?
ii. I: Who is the client?
1. The association is the client, the member is not
a. Can bring an action
iii. Westinghouse Exception (p. 268)
1. If the firm held itself out as independent counsel to individual members, communicates directly
with the members, promised the members confidentiality then if a reasonable belief in the
members is created an attorney-client privilege exists
a. Firm would be disqualified from representing adverse interest
f. Problem 3.4(d)
i. Discotrade Ltd. v. Wyeth-Ayerst International Inc. (p. 269)
1. I: Is the firm violating undivided loyalty to the client?
2. Can the firm represent Disco against WAII, when WAII’s sister corporation is a current client?
a. There is a duty of undivided loyalty
b. Burden of Proof:
i. The attorney seeking to represent an adverse interest has the burden of
demonstrating no actual or apparent conflict or no diminution of vigor in
representation
c. Court looks at factors:
i. Same or Overlapping
ii. Board
iii. Officers
iv. Legal Department
1. Very important because of the confidentiality issue
v. Systems, email, computer, support
vi. Trademarks, letterheads
d. Court held that companies are so closely related that the representation of one, means
representation of all, defendant fails to meet the burden of proof
3. May the firm represent ACC in an action against Wilson Disk, a sibling corporation of Johnson
Control, a client of the firm?
a. McCourt [p.273]; Stratagem [p.273]
b. Per se rule, a conflict with any member of a corporate group is a conflict with each
member
c. Contra – alter ego test
i. Modern test:
1. The standard expression of the showing one must make to prevail on
alter ego claim is that “(1) there is such a unity of interest that the
separate personalities of the corporations no longer exist; and (2)
inequitable results will follow if the corporate separateness is respected
4. Is a Per se rule the best approach?
a. ABA Formal Opinion 95-390
b. Facts & Circumstances Test
i. ABA
c. Restatement sec. 26
d. client’s” reasonable expectations
g. Problem 3.4(e)
i. Firm represents compuserve in the 6th circuit on FLSA, the firm represents other clients at DC level, firm
want to make arguments at appellate level
1. Is the firm disqualified from representing comperservices.com in the 6th Circuit if it is advocating
contrary positions for its other clients in District Court?
a. This is a potential positional conflict
i. A firm taking opposing legal positions on behalf of different clients in different
courts.
b. Firm is not disqualified because there is only a potential conflict
i. If it were in the same court or in a court of equal authority in another jurisdiction
there is an actual conflict, and informed consent would be needed
ii. Distinguish between potential and actual conflicts
1. Potential – All right ethically
35
2. Actual Conflicts – Need informed consent
h. RULE 1.9
i. Rule 1.9(a)
1. Without informed consent cannot represent another person in the same or substantially related
matter in which the matters are materially adverse
2. Ask first:
a. Are they materially adverse positions?
b. Are the substantially related
ii. Rule 1.9(b)
1. The problem of the moving lawyer
2. Ask:
a. Are the interest materially adverse to the previous client?
b. Are they substantially related?
c. Has the attorney acquired information protected by R 1.6 or R 1.9(c) that is material to
the matter
iii. Rule 1.9(c)
1. Confidentiality of information
2. Information of a former client shall not be used to the disadvantage of the former client in the
current matter
3. Lawyer shall not reveal information relating to the representation except as the rules permit
i. Problem 3.5
i. “substantially related?”
1. R 1.9 Comment 3
a. Same transaction or legal dispute, or
b. Substantial risk that information from former representation would help the current client
ii. They are substantially related in this problem  Representation inappropriate
j. Problem 3.6 Imputation of Disqualification
i. R 1.9(b)
1. Moving lawyer
2. Did she receive confidential information at the old firm relating to this representation?
ii. Assume Reynor actively worked on matters in which new firm represented adverse parties—
1. Attorney would be disqualified and the firm would be under R 1.10(a)
iii. When can we assume that the lawyer received confidential information?
1. To determine whether a lawyer might have acquired relevant confidences, a court should consider
whether, “(a) the lawyer and the client ought to have talked about particular facts during the
course of the representation, or (b) the information is of such character that it would not have been
unusual for it to have been discussed between lawyer and client during their relationship.”
iv. Firms cannot screen new attorneys to avoid this conflict
v. Firms can screen paralegals
1. Effective screen: R 1.0(k)
a. Written instructions to all employees:
b. (1) Prohibiting of the discussion of sensitive matters, keys, access codes, etc.;
c. (2) Restricting circulation of sensitive documents;
d. (3) Restricting access to files;
e. (4) Strong firm policy against breach, including affidavits of compliance and sanctions
for breach;
f. (5) Implementation of screen before conflicted lawyer begins service at new firm or
before firm accepted case creating the conflict.
g. R.1.9(b) – informed consent [for screening]
2. Imputed Disqualification
a. Firm – R.1.0(c)
b. R.1.10
i. A lawyer’s disqualification from handling a matter is imputed to all the
members of her firm.
ii. Lawyers in the same firm have access to confidential information and share
common financial interests
c. R.1.11
i. Former government attorney
ii. Screening allowed
d. R.1.18
36
i. Prospective clients
ii. Screening allowed
3. Changing Firms
a. Attorney is disqualified if:
i. Received confidential information related to the matter
ii. Matters are substantially related
b. Even if yes to just i. would probably be inappropriate under R 1.9(c)
k. Problem 3.7 – Advocate – Witness Conflict
i. Will the firm be disqualified because a partner is on the list of witness
1. R3.7
a. Is the partner’s testimony necessary, R 3.7(a)?
i. Lawyer cannot advocate unless
1. The testimony relates to an uncontested issue
2. The testimony related o the nature and value of legal services
3. Disqualification of the lawyer would work substantial hardship on the
client
b. Is the testimony helpful or prejudice?
i. Favorable – attorney disqualified
ii. Prejudicial – Firm and attorney disqualified
l. HYPO:
i. Does the firm have a duty to inform the client of the potential malpractice?
1. R.1.4 - Communications
2. Comment 7
a. A lawyer may withhold this information when:
b. When the client may respond imprudently, the lawyer may time the disclosure to
minimize that possibility
c. Pursuant to Court order
3. However, the lawyer may not withhold information to protect the lawyers interests or the interests
of others
m. Problem 3.8 Representation of Multiple Plaintiffs in Tort Case
i. Judge has no obligation to inquire in a civil case
ii. Assuming informed consent and waived claims, is representation appropriate?
1. No, brother could sue for contributory negligence and brother cannot waive the insurance
company’s claims
a. Would fail R 1.7(b)
n. Problem 3.9 Insurance Defense Practice
i. Who is the client?
1. Additional obligations on insurance lawyer to protect
2. Most states view the insured as the client, not the company
ii. Insurance attorneys will separate issues into those which they can handle and those in which other counsel
should be or is needed
iii. Question:
1. Whether the company is represented or just paying
a. If represented no confidentiality between attorney and insured
b. If just paying there is confidentiality between the attorney and insured (R 1.8 applies)
iv. Multiple representation requires full disclosure
o. Problem 3.10 Family Practice
i. Under common law divorce was adversarial therefore no multiple representation allowed
ii. Now with no-fault divorce this may not apply
1. Only adverse if arguing about something
iii. Attorney engaged in R 1.7(b) analysis to determine if he can represent husband and wife
1. Cannot be direct adversity (can’t have claims against each other)
2. Attorney’s reasonable belief that multiple representation is appropriate
a. Factors to consider in a divorce
i. The Need for Advocacy
1. Do the clients understand and have they resolved potential competing
claims
ii. The Need for Independent Counsel
1. Do the parties have emotional independence

37
iii. The lawyers ability to be neutral
Rule 1.3 Diligence
A lawyer shall act with reasonable diligence and promptness in representing a client.

Rule 2.1: Advisor


In representing a client, a lawyer shall exercise independent professional’s judgment and render candid advice. In rendering advice, a lawyer may refer not only to law
but to other considerations such as moral, economic, social and political factors, that may be relevant to the client’s situation.

Rule 3.1: Meritorious Claims and Contentions


A lawyer shall not bring or defend a proceeding, or assert or controvert an issue therein, unless there is a basis in law and fact for doing so that is not frivolous, which
includes a good faith argument for an extension, modification or reversal of existing law. A lawyer for the defendant in a criminal proceeding, or the respondent in a
proceeding that could result in incarceration, may nevertheless so defend the proceeding as to require that every element of the case be established.

Rule 3.4: Fairness to Opposing Party and Counsel


A lawyer shall not:
(a.) unlawfully obstruct another party’s access to evidence or unlawfully alter, destroy or conceal a document or other material having potential evidentiary value. A
lawyer shall not counsel or assist another person to do any such act;
(b.) falsify evidence, counsel or assist a witness to testify falsely, or offer an inducement to a witness that is prohibited by law;
(c.) knowingly disobey an obligation under the rules of a tribunal, except for an open refusal based on an assertion that no valid obligation exists;
(d.) in pretrial procedure, make a frivolous discovery request or fail to make reasonably diligent effort to comply with a legally proper discovery request by an opposing
party.
(e.) in trial, allude to any matter that the lawyer does not reasonably believe is relevant or that will not be supported by admissible evidence, assert personal knowledge
of facts in issue except when testifying as a witness, or state a personal opinion as to the justness of a cause, the credibility of a witness, the culpability of a civil litigant
or the guilt or innocence of an accused; or
(f.) request a person other than a client to refrain from voluntarily giving relevant information to another party unless:
(1.) the person is a relative or an employee or other agent of a client; and
(2.) the lawyer reasonably believes that the person’s interest will not be adversely affected by refraining from giving such
information.

Rule 4.1: Truthfulness in Statement to Others


In the course of representing a client a lawyer shall not knowingly:
(a.) make a false statement of material fact or law to a third person; or
(b.) fail to disclose a material fact when disclosure is necessary to avoid assisting a criminal or fraudulent act by a client, unless disclosure is prohibit by Rule 1.6

Rule 4.2: Communication with Person Represented by Counsel


In representing a client, a lawyer shall not communicate about the subject of the representation with a person the lawyer knows to be represented by another lawyer in
the matter, unless the lawyer has the consent of the other lawyer or is authorized to do so by law or a court order.

Rule 8.4: Misconduct


It is professional misconduct for a lawyer to:
(a.) violate or attempt to violate the Rules of Professional Conduct, knowingly assist or induce another to do so or do so through the acts of another;
(b.)commit a criminal act that reflects adversely on the lawyer’s honesty, trustworthiness or fitness as a lawyer in other respects;
(c.) engage in conduct involving dishonesty, fraud, deceit or misrepresentation;
(d.) engage in conduct that is prejudicial to the administration of justice;
(e.) state or imply an ability to influence improperly a government agency or official or to achieve results by means that violate the Rules of Professional Conduct or
other law; or
(f.) knowingly assist a judge or judicial officer in conduct that is a violation of applicable rules of judicial conduct or to other law.

CHAPTER 4: Ethical Issues in Civil Litigation: Limitation on Zealous Representation, Alternative Dispute Resolution, and Delivery
of Legal Services

I. Intro:
a. R 3.1 Meritorious Claims
i. Claims being brought must not be frivolous
1. Frivolous Claim:
a. A frivolous position is one that cannot be supported by good faith factual and/or legal
arguments.
b. A frivolous position is one that a lawyer of ordinary competence would recognize as so
lacking in merit that there is no substantial possibility that the tribunal would accept it.
[Restatement]
c. An action is not frivolous even though the lawyer believes the client’s position will
ultimately fail.
ii. FRCP 11
1. Attorney’s responsibility
a. Facts –

38
i. must conduct a reasonable inquiry to make sure that the action is not being filed
for an improper purpose [harass, cause unnecessary delay]
b. Law –
i. must determine if legal contentions are warranted by existing law or by non
frivolous arguments for change.
c. Evidence –
i. must determine if there is evidence to support the matters to be alleged in the
complaint or if evidentiary support is likely to be identified in discovery.
II. Problem 4.1 Frivolous Claims
a. 4.1(a)
i. Duties prior to commencing civil litigation
1. Is the document frivolous?
2. Are the legal contentions warranted?
3. Is there evidentiary support for contentions or evidence (or for denying adversaries contentions)?
ii. HYPO:
1. Attorney files complaint alleging plaintiff was injured by the type of music that was played during
her surgery; the surgeon played heavy metal music and that damaged the nerves of her back and
prevented complete recovery.
a. What can the attorney who wants to advocate a novel theory do to minimize the risk of
sanctions for filing a frivolous action?
i. Conduct an investigation to determine if evidence and facts support
1. Memorialize in Pre filing research memorandum
b. 4.1(b)
i. R 1.3
1. Requires a good faith basis
2. Even if the claim has a good faith basis, but the clients interest is to embarrass (FRCP 11 still
requires that claims be brought for a proper purpose
a. Analysis:
i. Will it result in a vindication of the client’s rights?
1. If yes, then proper
2. If primarily results in the imposition of cost on the defendant, improper
ii. …or simply burden the other side?
ii. R 2.1 Counseling
1. Convince client to not bring frivolous claim by counseling
a. Active listening (i.e. “You want to go to Virginia and fight in the MMA?”)
b. Suggest counseling
2. What if client insists on brining this claim?
III. Problem 4-2
a. Was it proper for the investigator to attempt to interview hotel employees
i. R 4.2 Comment 7
1. I: Are the employees represented?
2. Probably cannot talk to kitchen employees because statements may be imputed to the hotel, may
constitute an admission
ii. Whose statements will constitute an admission of the organization?
1. Managerial employees
2. Is the employee authorized to speak on behalf of the organization [State Law]
3. Is the statement about a matter within the scope of the employee's duties [Federal Law R.801]
iii. Was it proper for the hotel attorney to order the employees to not talk to anyone about the case?
1. R 3.4(f)
a. Improper to order, could request
iv. What if the hotel sent a letter to the employees telling them to not talk to anyone outside, only the hotel’s
attorney
1. R 3.4(f) does not apply, rules do not apply to hotel managers
v. What if the hotel’s attorney advised the hotel to send the letter?
1. R 8.4(a) prohibits, attorneys cannot get someone else to do something they cannot do
vi. Attorneys can talk to former employees unless seeking to obtain privileged information (R 4.4)
***
ADVERTISING
Rule 7.1: Communications Concerning a Lawyer’s Services

39
A lawyer shall not make a false or misleading communication about the lawyer or the lawyer’s services. A communication is false or misleading if it contains material
misrepresentation of fact or law, or omits a fact necessary to make the statement considered as a whole not materially misleading

Rule 7.2: Advertising


(a.) Subject to the requirements of Rules 7.1 and 7.3, a lawyer may advertise services through written, recorded or electronic communication including public media.
(b.) A lawyer shall not give anything of value to a person for recommending the lawyer’s services except that a lawyer may
(1.) pay the reasonable costs of advertisements or communications permitted by thus Rule;
(2.) pay the usual charges of a legal service plan or a not-for-profit or qualified lawyer referral service. A qualified lawyer
referral service is a lawyer referral service that has been approved by an appropriate regulatory authority;
(3.) pay for a law practice in accordance with Rule 1.17; and
(4.) refer clients to another lawyer or a nonlawyer professional pursuant to an agreement not otherwise prohibited under these
Rules that provides for the other person to refer client or customer to the lawyer, if
(i.) the reciprocal referral agreement is not exclusive, and
(ii.) the client is informed of the existence and nature of the agreement
(c.) Any communication made pursuant to this rule shall include the name and office address of at least one lawyer or law firm responsible for its content.

Rule 7.3: Direct Contact with Prospective Clients


(a.) A lawyer shall not by in-person, live telephone or real-time electronic contact solicit professional employment from a prospective client when a significant motive
for the lawyer’s doing so it the lawyer’s pecuniary gain, unless the person contact:
(1.) is a lawyer; or
(2.) has a family, close personal, or prior professional relationship with the lawyer
(b.) A lawyer shall not solicit professional employment from a prospective client by written, recorded or electronic communication or by in-person, telephone or real-
time electronic contact even when not otherwise prohibited by paragraph (a), if:
(1.) the prospective client has made known to the lawyer a desire not to be solicited by the lawyer; or
(2.) the solicitation involves coercion, duress or harassment
(c.) Every written, recorded or electronic communication from a lawyer soliciting professional employment from a prospective client known to be in need of legal
services in a particular matter shall include the words “Advertising Material” on the outside of the envelope, if any, and at the beginning and ending of any recording or
electronic communication, unless the recipient of the communication is a person specified in paragraphs (a)(1) or (a)(2).
(d.) Notwithstanding the prohibitions in paragraph (a), a lawyer may participate with a prepaid or group legal service plan operated by an organization not owned or
directed by the lawyer that uses in-person or telephone contact to solicit membership or subscription for the plan from persons who are not known to need legal services
in a particular matter covered by the plan.
a. R 7.1
vii. Is the information truthful?
viii. Are they misleading?
ix. Are the images they convey good for the profession?
b. Is the information useful in helping consumers find an attorney or does it disparage a particular group
c. R 7.2
x. R 7.2(b)
1. Lawyer may not give anything of value to a person for recommending the lawyers services
a. BUT, lawyers may have mutual referral agreements (but need to inform the client of the
reciprocal agreement)
d. R 7.3
xi. R 7.3(a)
1. Non-solicitation
a. Exceptions:
i. Family
ii. Friends
iii. Anyone you have a close relationship with
xii. R 7.3(c)
1. Mail and electronic advertisements must be labeled “advertisement”
Rule 1.8: Conflict of Interest: Current Clients: Specific Rules
(a.) A lawyer shall not enter into a business transaction with a client or knowingly acquire an ownership, possessory, security or other pecuniary interest adverse to a
client unless:
(1.) the transaction and terms on which the lawyer acquires the interest are fair and reasonable to the client and are fully
disclosed and transmitted in writing in a manner that can be reasonably understood by the client;
(2.) the client is advised in writing of the desirability of seeking and is given a reasonable opportunity to seek the advice of
independent legal counsel on the transaction; and
(3.) the client gives informed consent, in a writing signed by the client, to the essential terms of the transaction, including
whether the lawyer is representing the client in the transaction.
(b.) A lawyer shall not use information relating to representation of a client to the disadvantage of the client unless the client gives informed consent, except as
permitted or required by these Rules.
(c.) A lawyer shall not solicit any substantial gift from a client, including a testamentary gift, or prepare on behalf of a client an instrument giving the lawyer or a person
related to the lawyer any substantial gift unless the lawyer or other recipient of the gift is related to the client. For purposes of this paragraph, related persons include a
spouse, child, grandchild, parent, grandparent, or other relative or individual with whom the lawyer or the client maintains a close, familial relationship.
(d.) Prior to the conclusion of representation of a client, a lawyer shall not make or negotiate an agreement giving the lawyer literary or media rights to a portrayal or
account based in substantial part on information relating to the representation.
(e.) A lawyer shall not provide financial assistance to a client in connection with the pending or contemplated litigation, except that:
(1.) a lawyer may advance court costs and expenses of litigation, the repayment of which may be contingent on the
outcome of the matter; and
(2.) a lawyer representing an indigent client may pay court costs and expenses on behalf of the client.

40
(f.) A lawyer shall not accept compensation for representing a client for one other than the client unless:
(1.) the client gives informed consent
(2.) there is no interference with the lawyer’s independence of professional judgment or with the client-lawyer relationship;
and
(3.) information relating to representation of a client is protected as required by Rule 1.6
(g.) A lawyer who represents two or more clients shall not participate in making an aggregate settlement of the claims of or against the clients, or in a criminal case an
aggregated agreement as to guilty or nolo contendere pleas, unless each client gives informed consent, in a writing signed by the client. The lawyer’s disclosure shall
include the existence and nature of all the claims or pleas involved and of the participation of each person in the settlement
(h.) A lawyer shall not:
(1.) make an agreement prospectively limiting the lawyer’s liability to a client for malpractice unless the client is
independently responsible in making the agreement; or
(2.) Settle a claim or potential claim for such liability with an unrepresented client or former client unless that person is
advised in writing of the desirability of seeking and is given a reasonable opportunity to seek the advice of independent legal counsel in connection
therewith.
(i.) A lawyer shall not acquire a proprietary interest in the cause of action or subject matter of litigation the lawyer is conducting for a client, except that a lawyer may:
(1.) acquire a lien authorized by law to secure the lawyer’s fee or expenses; and
(2.) contract with a client for a reasonable contingent fee in a civil case.
(j.) A lawyer shall not have sexual relations with a client unless, a consensual sexual relationship existed between them when the client-lawyer relationship commenced
(k.) While lawyers are associated in a firm, a prohibition in the foregoing paragraphs (a) through (i) that applies to any one of them, shall apply to all of them.

RULE 1.8
I. Rule 1.8 deal with the categories of conflicting interests that are prohibited irrespective of the analyses we might perform
pursuant to Rules 1.9 and 1.8.
II. Some of these conflicts will also be subject to Rule 1.10 Imputation, generally, the ones that are not considered “personal”
conflicts.
III. The purpose of these rules is to:
a. Protect the integrity of the lawyer-client relationship;
b. Maintain an appropriate balance of power between the lawyer and the client, i.e, insure that the lawyer is always
acting in a fiduciary capacity with respect to the clients interests; that the lawyer never engages in activities that
would encourage the lawyer to place her interests above the clients;
c. Protect clients during periods of vulnerability.
d. The rules should be interpreted with these policies in mind.
IV. RULE 1.8:
a. R.1.8(a) – Business transactions with clients
i. Business relationships [any business interest or transaction] with clients are prohibited unless:
ii. [Business relationships other than an attorney-client relationship]
iii. The terms are fair and reasonable, there is full disclosure, everything is in writing, and the client fully
understands the terms of the transaction
iv. The client has been advised in writing to seek independent counsel
v. The client provides informed consent
vi. QUESTION:
1. One year ago at the request of Investor, Attorney incorporated Sundown, Inc., a corporation
organized for the purpose of developing a shopping mall. Investor is the president and sole
shareholder of Sundown. Attorney is Sundown’s counsel of record. Sundown has run out of
funds and needs $150,000 to complete the construction of a parking lot. Investor has asked
Attorney to lend Sundown $150,000 at the current normal rate of interest. Instead, Attorney
persuades Investor to let Attorney purchase $150,000 worth of stock in Sundown. The transaction
is fair and reasonable.
a. (a) Attorney’s conduct is proper because the Rules of Professional Conduct do not
govern a lawyer’s business activities.
b. (b) Attorney’s conduct is proper because Attorney has not acquired a proprietary
interest in Sundown’s claims.
c. (c) Attorney’s conduct is proper unless Attorney represents Sundown in the stock
purchase transaction.
d. (d) Attorney’s conduct is proper if Attorney advises Sundown in writing to seek the
advice of independent counsel on the transaction.
i. CORRECT
b. R.1.8(b) – Using client information to client’s disadvantage
i. Larry Lawyer represents Clara Client in Clara’s efforts to purchase the WaterWorld Amusement Park from
WWA Associates, Inc. After the commencement of the representation Larry purchases 100 share of WWA
Associates stock from Marvin, a member of Larry’s bowling league.
1. (a) Larry’s behavior is permissible under the Bioron exception.

41
2. (b) Larry’s behavior is permissible if Larry does not believe that there is any material
limitation on his ability to represent Clara.
3. (c) Larry’s behavior is not permissible because Marvin has not provided his informed
consent.
4. (d) Larry’s behavior is not permissible.
a. [Larry has used confidential information for his personal benefit and has become a
potential seller of the shares that his client is attempting to acquire, creating a
concurrent conflict between himself and the client.]
c. R.1.8(c) – Client gifts to lawyers
i. A lawyer may not prepare any document which provides a gift or bequest to the lawyer or to a person
related to the lawyer
1. Exception
a. Lawyer prepares an instrument for a family member
ii. Your long standing client wants to remember you in her will, can another member of your firm prepare it?
1. R.1.8(k) – Imputation of prohibition
iii. Clifton Counsel has represented Sylvia Stone, his father’s second wife for many years. Recently Sylvia
asked Counsel to revise her will and to include a substantial bequest to Counsel’s daughter to further her
education.
1. (a) Counsel may not allow Sylvia to make a substantial bequest to his daughter.
2. (b) Counsel may prepare Sylvia’s revised will to provide the bequest.
a. CORRECT
3. (c) Counsel may not prepare the revised will but a member of his firm may do so.
4. (d) Counsel may prepare the will only if it includes an insignificant bequest to his daughter.
d. R.1.8(d) – Literary or media rights
i. Literary and Media Rights [prior to the conclusion of representation]
1. Prohibited
ii. Lois Lawyer is defending Wanda in a trial for the murder of Wanda’s ex-husband. The judge has denied
the Media’s request that the trial be televised. Fox News has offered Lois a substantial sum if she will wear
a concealed microphone to the proceedings, the transcript of which will not be released until the trial has
concluded. Wanda has provided her informed consent to this agreement.
1. (a) Lois may enter into the contract with Fox News.
2. (b) Lois may only enter into the contract with Fox News if Wanda receives 33% of the fee.
3. (c) Lois may not enter into the contract with Fox News because it would constitute ineffective
assistance of counsel.
4. (d) Lois may not enter into the contract with Fox News because it creates a concurrent
conflict.
a. CORRECT
e. R.1.8(e) – Financial assistance to clients
i. Other than advancing court costs and litigation expenses, an attorney may not subsidize a client’s living
expenses
f. R.1.8(f) – Payment of fees by non client
i. Client gives informed consent
ii. Paying party does not interfere with lawyer’s profession judgment or with the lawyer-client relationship
iii. Confidentiality is maintained
g. R.1.8(g) – Aggregate settlements
i. The lawyer may not accept an settlement on behalf of multiple clients unless each is fully informed of the
terms and agrees, giving informed consent in writing.
h. R.1.8(h) – Prospective limitations of malpractice liability
i. Prospective limitations of Malpractice liability, settling malpractice claims of unrepresented client or
former client [must advise in writing of the desirability of seeking counsel]
1. Exceptions
a. Client [sophisticated] independently represented by counsel
b. Arbitration agreements
i. R.1.8(i) – Proprietary interests in client claims
i. Proprietary interests in claims or litigation
ii. Creates conflicts of interest between with lawyer and the client; e.g., the lawyers interest in the matter
would enable her to participate in establishing the goals of the representation
iii. Difficulty in firing the attorney if the lawyer owns part of the claim

42
1. Exceptions
a. Liens - which give the attorney a right to payment from the proceeds of the client’s claim
b. Contingent fees
j. R.1.8(j) – Sexual relations with clients
i. Prohibited unless the relationship predates the representation.
ii. Jackie Juarez is a divorce attorney. One of her clients, Walt Waters, a multi millionaire rock star and film
idol, has fallen in love with Jackie.
1. (a) Jackie may not have a sexual relationship with Walt until Walt’s divorce is final.
2. (b) Jackie may begin a sexual relationship with Walt as long as there is no interference with
her professional judgment and as long as she keeps the client’s information confidential.
3. (c) Jackie may not have a sexual relationship with Walt.
a. CORRECT
4. (d) Jackie may begin a sexual relationship with Walt if she provides notice to her firm and
the opposing party.
k. R.1.8(k) – Imputation of Prohibitions
i. Imputation of Prohibitions
ii. Generally, all R.1.8 conflicts will be imputed to members of the lawyer’s firm or practice group.
1. Exception
2. R. 1.8(j) is personal and is not imputed to other lawyers in the firm

43

Das könnte Ihnen auch gefallen